27 авг. 2009 г.

Разбор неопределённости (задача о двух конвертах)

Старинный анекдот звучит так:
- Какова вероятность встретить на центральной площади динозавра?
- Одна вторая! Или встречу, или не встречу.

Главный смысл, который не все извлекают из этого анекдота, состоит в следующем: если у дающего ответ недостаточно информации, то это не значит, что разумно будет считать все возможные варианты равновероятными.

Поясню:
- если речь идёт о центральной площади Планеты динозавров, то вероятность становится очень большой - там почти всегда гуляют динозавры,
- если речь идёт о центральной площади Планеты роботов, то вероятность крайне мала - динозавры там просто не могут возникнуть из-за отсутствия атмосферы (разве только кто-то привезёт в аквариуме динозавра с Планеты динозавров).

Вернёмся к парадоксу двух конвертов. Напомню, что у нас есть два конверта, в одном из которых находится вдвое большая сумма, чем в другом. Допустим, мы открыли конверт, в котором ровно 10 денег. Далее следует обман, который не так легко разглядеть - авторы статьи говорят: «Стало быть, в другом конверте лежат либо $5, либо $20 с вероятностью 50 х 50».

Хочется читать статью дальше, потому что это утверждение кажется очевидным. Но надо остановиться и подумать, а не герои ли мы анекдота. Только что мы специально доказали, что все лошади одного цвета (более того, чтобы жизнь мёдом не казалась, мы надёжно установили, что это лиловый цвет). Там было очевидно, что в доказательстве есть обман (почти каждый человек видел лошадей разных цветов). В этой же задачке всё не так очевидно, потому что вокруг много магических слов: «редукция кота Шрёдингера», «свыше 20 миллионов компьютерных симуляций», «Броуновский храповик», «Накачка волатильности»... (вспоминается разговор о массовой утере культуры научно-популярной литературы). Поэтому суть обмана заметить не так легко. Но вполне возможно.

Представьте себе, что Якубович каждый раз кладёт в конверты 10 и 20 рублей. Если в выбранном конверте 10 рублей, то с какой вероятностью во втором будет 5, а с какой 20? Ясно, что во втором с вероятностью 100% будет 20 рублей (иначе быть не могло).

Если Якубович кладёт в первый конверт случайную сумму от 1 до 100 рублей, а во второй - в два раза больше, то всё будет иначе: если в выбранном конверте больше 100 рублей, то во втором абсолютно точно будет в два раза меньше (опять никаких 50х50 не получается). А вот если меньше или ровно 100, то имеет смысл попробовать взять второй конверт - так и впрямь будет выгоднее :)

Кто-то скажет, что принцип укладывания денег в конверты не был оговорён в условии, было известно только, что в одном конверте сумма в два раза больше, чем в другом. Но ведь об этом и речь! Мы не можем знать, с какой вероятностью во втором конверте удвоенная сумма, а с какой - её половина, поэтому и не можем этим пользоваться. А если использовать такие недостоверные представления о вероятностях, то можно что угодно доказать! И потом можно будет смело называть это парадоксом своего имени. Кстати, заметьте, что в статье постоянно ссылаются именно на британского учёного, как бы намекая, что уши развешивать не надо.

Дополнение: заметка вызвала множество вопросов в комментариях, поэтому я решил дополнить её ссылкой на статью в википедии (пока рекомендую только английскую версию, поскольку в русском варианте соответствующей статьи тоже нагнетается атмосфера загадочности). Предлагаю изучить предложенные в википедии объяснения ловушки - там предлагается посмотреть на неё с разных сторон. Надеюсь, альтернативные взгляды на проблему помогут разрешить внутренние противоречия между интуицией и научным знанием.

Дополнение2: В русскоязычном разделе википедии тоже появился качественный материал на эту тему, поэтому рекомендую его всем, желающим разобраться в проблеме.


Что в этом всём важно? А то, что очень мало людей имеют квалификацию, чтобы замечать такие логические ловушки. В данный момент вовсю идёт интеграция элементов теории вероятностей в школьные курсы, но где взять учителей, понимающих вероятностные тонкости на достаточном уровне? Мы говорили про это недавно, но перед самым началом учебного года есть желание повториться. Многие ли учителя, увидев обсуждаемую статью, содержащую откровенный бред, не принесут её в школу? У них же есть правильное и естественное желание - поделиться самым лучшим с талантливыми школьниками (чтобы интерес к предмету поддержать). А результат этого известен заранее - запутанные мозги, неправильные представления о красивой и стройной науке, ощущение, что «это всё шаманство какое-то» («это нельзя понять, поэтому просто запоминайте, дети!»). Если школьник заподозрит, что в предложенной статье что-то не то, то учитель не сможет ему объяснить, в чём проблема, потому что сам этого не понимает.

Учить серьёзной науке - это вовсе не пересказывать главы учебника своими словами. В теории вероятностей очень сложные и очень простые задачи могут формулироваться одинаково маленьким количеством слов. И надо иметь серьёзную квалификацию, чтобы молниеносно различать эти случаи. Желание учить основам теории вероятностей в школе понятно - статистика всюду вокруг нас, поэтому надо её понимать. Но очень не хочется получить усиление каши в головах из-за такого нововведения. Пока нету массовой культуры решения задач по комбинаторике (а её, увы, нету даже среди учителей, в чём легко убедиться, присутствуя при проверке олимпиад уровня города и ниже), наивно надеяться, что эти же преподаватели смогут освоить теорию вероятностей (а это необходимо, чтобы потом учить детей).

Нет, эти преподаватели вовсе не плохие, они могут квалифицированно разобраться со многими типами задачек. Но если их самих никогда толком этому не учили, то и они не смогут справиться с простым олимпиадным вопросом. Первый шаг - массовое обучение учителей. Потом эти преподаватели будут передавать свои знания школьникам. И только потом можно включать вопросы теории вероятностей в ЕГЭ (второй раз уже рекомендую более подробные размышления fdo об этой теме). Если я правильно понимаю, то сейчас план противоположный - расширить список тем ЕГЭ, а ученики и учителя пусть выкручиваются, как умеют. Будем надеяться, всё пройдёт лучше.

Хорошего вам дня и спасибо, если найдёте возможность порекомендовать эту заметку в своём блоге.

258 комментариев:

  1. "Нужно менять или не менять конверты в каждом заходе случайным образом, но с вероятностью, которая зависит от суммы, увиденной в первом конверте. То есть чем меньше сумма в конверте А, тем с большей вероятностью следует сменить конверт и наоборот, несколько большая сумма в А говорит о том, что скорее следует оставить первый конверт себе."
    Цитата из статьи "http://www.membrana.ru/articles/simply/2009/08/19/174500.html".
    Была ли она в оригинале?
    Если да, то никакого обмана здесь нет. Наличие максимальной (пусть и неизвестной) суммы порождает "неравновероятную" задачу. Использование приведенного алгоритма позволяет на этом сыграть.

    ОтветитьУдалить
  2. Basilvs, обман в первой части статьи, где обосновывается мысль о том, что всегда выгоднее менять первый конверт на второй (цитата: «То есть в общем виде, если в конверте А лежит сумма С, то статистически ожидаемая сумма в конверте B составит 0,5 х С/2 + 0,5 х 2С = 5/4 С. Таким образом, теория говорит, всегда выгодно менять первоначальный свой выбор (12,5 больше 10), хотя в отдельных раундах вы будете проигрывать.»).

    После этого нам ещё раз проезжают по ушам фразой «математическое сообщество до сих пор не пришло к консенсусу, так что задача осталась открытой». Это же ерунда! Красиво оформленная, но ерунда.

    Ну а далее всё как раз разумно: говорится, что если смену конверта проводить только для некоторых случаев (когда сумма меньше определённой величины), то удаётся заработать больше. Это как раз вполне естественно, логично и легко обосновывается. Но стоило ли запутывать и обманывать читателя в первой части, чтобы потом показать пустую и очевидную банальность?

    Польза статьи только в том, что некоторые читатели разобрались в этом трюке с подменой одной вероятности на другую.

    ОтветитьУдалить
  3. Давай тогда формулировать так: обман в этом самом утверждении.
    Статья же никого не обманывает, а просто рассматривает несколько подходов к проблеме. Причем подходы эти упорядочены по нарастанию корректности :)

    ОтветитьУдалить
  4. Basilevs, в комментарии выше я процитировал откровенную ложь из статьи. Некорректных с точки зрения математики высказываний там тоже хватает. Эти строки, конечно, можно называть "не обманом, а просто одним из подходов к проблеме", но кому от этого будет хорошо?

    Да, в статье есть несколько разумных мыслей, но есть и очень много откровенного непрофессионализма или намеренного введения в заблуждение. Формирование мифа о том, что метод "всегда бери другой конверт" выгоден, запутывание читателей, желающих разобраться в проблеме - это главные минусы статьи.

    Basilevs, для Вас, скорее всего, было очевидно, где в статье бред, поэтому Вы его легко пропустили мимо ушей. Но это не означает, что статья корректна.

    ОтветитьУдалить
  5. Задача на арифметику.

    Выбирая второй конверт мы просто делаем ставку по шансам банка. Для любого знакомого с покером это понятно. Например, наша вероятность победить в какой-то момент игры может быть 30%, но мы делаем ставку Х, потому что в случае победы мы получим 5Х)

    Так и здесь, вероятность победы (выбора большей суммы) по-прежнему 50%, но с этой вероятностью мы выигрываем больше, чем можем проиграть.
    Потерять мы можем только 0.5Х, а выиграть не 1.5х, а 2Х.
    Т.е. корректное условие: либо удвоение, либо ничего, как вариант - либо 0.5Х либо 1.5Х.

    Так что ваша фраза: "Мы не можем знать, с какой вероятностью во втором конверте удвоенная сумма, а с какой - её половина, поэтому и не можем этим пользоваться." в данном случае ничего не объясняет и не нужна для объяснения парадокса.

    Что бы было совсем просто, представьте, что сумма либо в 100 раз больше, либо в 100 раз меньше. (Вместо 2-х). Открыли конверт- увидели там 100 рублей. Теперь предпочтение другого означает, что вы ставите 99 рублей с возможностью выиграть 10000.
    Вероятность то 0.5, а денег?

    Или я что-то все-таки не понял?

    ОтветитьУдалить
  6. Jekyll, вероятность не 50%.

    ОтветитьУдалить
  7. Jekyll, Вы всё верно говорите про отношение вероятности выиграть к размеру выигрыша.

    Но ошибка у Вас в предположении, что вероятности выигрыша и проигрыша одинаковы (Ваши слова «Вероятность то 0.5, а денег?» ведь об этом?).

    В реальности же эту вероятность мы не знаем.

    Давайте в Вашем примере (суммы относятся в 100 раз) проанализируем такую ситуацию: Якубович всегда кладёт в один конверт 1 рубла, а в другой - 100 рублей.

    Если Вам достался конверт с сотней, то во втором 100% будет 1 рубль. И надеяться, что удастся выиграть 10000 уже не надо (даже если игрок не знает этого принципа распределения сумм).

    Человек может думать, что он ставит 99 рублей, чтобы выиграть 10000, но в реальности это может быть совсем не так. И именно недоопределённость задачи позволяет махинировать с вероятностями, серьёзно сбивая с толку :)

    ОтветитьУдалить
  8. Анонимный28.08.2009, 15:36

    А почему вы так акцентируете на Якубовиче, максимальной возможной сумме "в реальности" и т.п?
    Задача ведь теоретическая, а не практическая.
    Т.е. давайте рассматривать не якубовича, а некий компьютер, который с вероятностью 50% во второй конверт кладет сумму больше, и 50% - меньше.
    В таком случае стратегия "всегда брать второй конверт" адекватна.

    ОтветитьУдалить
  9. Уважаемый аноним, в самом деле, разбирая эту задачу, я концентрируюсь на реальности. Если бы нам хотелось верить в какой-то мифический мир, где вероятности двух взаимоисключающих событий всегда одинаковы, то можно смело забыть о математике и верить, что брать другой конверт выгоднее.

    Но если Вас интересует именно настоящая и честная ситуация, то рекомендую вникнуть в то, что нет никакой разницы, кто будет раскладывать деньги по конвертам: человек или робот. Стратегия "всегда бери другой конверт" не является стратегией и не является выигрышной. Это вообще бредовый домысел.

    И я надеялся, что достаточно внятно написал об этом в данной заметке. Но, если у Вас остались вопросы, то можем обсудить подробнее. Если хотите, то можно по электронной почте (мой адрес - mytribune AT yandex.ru). Удачи!

    ОтветитьУдалить
  10. Анонимный28.08.2009, 16:01

    Если я не ошибаюсь, то при отсутствии априорной информации, по принципу максимума энтропии будет вполне законным будет предположить равномерное распределение. То есть, если нам не сказали, где мы находимся или какую-то дополнительную информацию о конвертах, то для нас вероятность обоих событий и будет 50/50

    ОтветитьУдалить
  11. Уважаемый аноним, означают ли Ваши слова, что Вы верите, что вероятность встретить динозавра на центральной площади Вашего города тоже 50/50?

    Полагаю, что нет. А раз так, то давайте и с этой задачкой разберёмся.

    Итак, в один конверт робот укладывает случайную сумму от 1 до N, а во второй - вдвое большую сумму. Это Ваша идея равномерного распределения, правильно?

    Раз так, то если мы достали конверт, в котором денег больше N, то вероятность того, что во втором конверте будет сумма в два раза больше равна нулю (так как это просто невозможно). И это так, даже если игрок не знает N. Поэтому никаких 50/50 не получается при таком подходе.

    Или Вы предлагаете другое равномерное распределение? Если да, то какое?

    ОтветитьУдалить
  12. Илья,
    Вы вводите дополнительные условия с повторением эксперимента и дополнительными знаниями("якубович всегда кладет..."), которых в оригинальной задаче нет.

    Представьте что я даю Вам сейчас два конверта и говорю, что в одном денег больше, чем в другом в два раза. Вы открываете один и видите 10 рублей. Будете ли Вы открывать второй?
    Давайте подумаем, почему Вы можете принять решение не открывать и что руководит Вами. Очевидно, утилиты действий фиксированы (-0.5, +1.5), значит выбор в люом случае диктуется вероятностью. Только вероятность эту следует понимать не в классическом смысле, а в субъективном! (de finneti, savage). Например, 10 рублей кажутся Вам отличным призом который Вы не хотите потерять(синица в руках), и поэтому Вы подсознательно выставите большую вероятность тому, что во втором конверте 5 рублей. Естественно, в такой ситуации не открывать уже более выгодная стратегия. Это и происходит когда "интуиция возражает"!
    Очевидно, что если напротив, Вы считаете, что 10 рублей, что 5 - никакой разницы, а вот 20 уже будет неплохо - Вы присвоите большую вероятность 20и, и откроете второй конверт.
    Вероятности таким образом равняются P(5)=q P(20)=1-q, где q это коэффициент того, насколько Вы склонны принимать риск.
    Очень хороший пример того, как это работает можно найти в обсуждениях парадоксов Allais и Ellsberg.

    ОтветитьУдалить
  13. Попробую объяснить, как ситуация выглядит со стороны.
    В статье на membrana.ru формулируется совершенно определенная задача. Ставится условия, приводится решение, дается четкий ответ. Вы же говорите, что при каких-либо других условиях приведенное решение неверно.

    Приведу абстрактный пример на пальцах.
    Автор статьи: найти решение уравнения x + 5 = 10. Ответ, который абсолютно логичен: 5.
    Вы же утверждаете, что автор не прав, аргументируя это тем, что в шестнадцатеричной системе счисления ответ вовсе не 5, а B. Разница - в подходе к постановке задачи: автор по умолчанию подразумевает, что дополнительных условий нет, раз о них не сказано особо, а вы по умолчанию считаете, что они есть, но неизвестны.

    Другой вопрос, что первый из этих подходов (математические, логические задачи) практически всегда используется в системе образования, а второй ("задачки с подвохом") чаще встречается в жизни и, соответственно, практически более значим.

    ОтветитьУдалить
  14. chester:

    1. если сумма ограничена сверху, срабатывает условная вероятность, о которой говорил Илья.

    2. Если сумма не ограничена сверху, то ожидание выигрыша при любой стратегии бесконечно большое, и сравнивать их не имеет смысла.

    ОтветитьУдалить
  15. bkonst:
    Ни о какой ограниченности в исходной постановке задачи речи не было.

    А в неограниченном случае, конечно, они набедокурили, но и Илья меня своими объяснениями запутал. Весь "парадокс" разрешается, если осознать, что случайная величина (дискретная) в данном случае - количество денег в изначально выбранном конверте.

    К моменту выбора конверты запечатаны, сумма денег фиксирована. Выбор идет между х и 2х. Если количество денег в изначально выбранном конверте - обозначить за "А", а нашу стратегию - факт смены конверта - за "Б", то возможны следующие варианты
    1) А = х, Б = true; Выигрыш - 2х;
    2) А = х, Б = false; Выигрыш - х;
    3) А = 2х, Б = true; Выигрыш - х;
    4) А = 2х, Б = false; Выигрыш - 2х;

    Вероятность исходов A = x и А = 2х одинакова и равна 1/2. Получаем, что при любом Б ожидаемый выигрыш равен 1/2*х + 1/2*2х = 1.5х.

    ОтветитьУдалить
  16. "Стратегия "всегда бери другой конверт" не является стратегией и не является выигрышной. Это вообще бредовый домысел." ?????
    Я, по-прежнему, не вижу изъяна в моей логике. Если, конечно, не апеллировать к мифическим Якубовичам.

    Задача поиска оптимальной стратегии выигрыша сводится здесь не к вероятности, а к тупой арифметике. Иллюзии возникают только из-за цифры 2.

    Даже не знаю, нужно ли что-то еще доказывать.

    ОтветитьУдалить
  17. chester: вы статью на мембране-то прочли? Вы сейчас сами противоречите утверждению "теория говорит, всегда выгодно менять первоначальный свой выбор" из неё.

    ОтветитьУдалить
  18. Поймите, подвох здесь не в вероятностях, а в том, что по условиям задачи нам обещают большее за меньшее.

    Не имело бы смысла менять конверты, если бы сумма увеличивалась в два раза(как в исходной задаче) либо становилась бы равной нулю(помним, что она не равна нулю, а всего лишь в 2 раза меньше по условию).

    Т.е. это арифметика:
    Пусть сущ. условие а): в конверте Х денег, выбирай, меняешь ли ты его при условии, что в другом конверте либо 2Х денег, либо 0. - тут менять смысла нет. Правильно? Ну, хочешь меняй, хочешь не меняй, не наживешься.

    Как же может быть верным ваше утверждение об отсутствие выигрышной стратегии в ситуации если выигрыш равен 2х, а проигрыш приводит уже не к потере всего, а только половины? Или а) не верно?

    Только не апеллируйте к пограничным Якубовичам.

    ОтветитьУдалить
  19. Jekyll: если в другом конверте в 9 случаях из 10 будет X/2 - менять невыгодно, так что тупая арифметика подсказывает неверно.

    Условия задачи не гарантируют одинаковой вероятности X*2 и X/2 в другом конверте.

    Хотите - промоделируйте сами.

    ОтветитьУдалить
  20. Jekyll


    Т.е. это арифметика:
    Пусть сущ. условие а): в конверте Х денег, выбирай, меняешь ли ты его при условии, что в другом конверте либо 2Х денег, либо 0. - тут менять смысла нет. Правильно? Ну, хочешь меняй, хочешь не меняй, не наживешься.

    неверно.

    Предположим, я выбираю менять, и там 2х, следовательно, я в выигрыше. С точки зрения классической теории решений, у меня есть два исхода
    не менять х*1 = 1х (1 - P(1)=1 - оставляем конверт в котором наверняка х)

    менять 0х*0.5+2х*0.5=1х

    то есть исходы равнопривлекательны. Фокус здесь в том, что вероятности с точки зрения принимающего решения не равняются 0.5. Проиллюстрируем это таким примером

    1. У Вас в руках 50 рублей. Выберите ли Вы играть до 100, если вероятность выиграть 1/2?

    2. У Вас в руках 1000000 рублей. Выберите ли Вы играть до 2000000, если вероятность выиграть 1/2?


    Я полагаю, что ответы разные. Почему и как это смоделировать?

    Здесь можно использовать два подхода - использовать субъективные вероятности(я писал выше) или изменять выигрыши. Расстройство от потери 1000000 очевидно гораздо выше, чем от потери 50 рублей, так что пример 2 преобразуется в

    2. -5х*0.5+1х*0.5=-4х (следовательно - не играть!)

    где -5х выражает объем Ваших страданий по поводу потери миллиона который был у Вас в руках. (см Allais paradox для подробного рассмотрения ситуации)

    ОтветитьУдалить
  21. Jekyll:
    Допустим, мы взяли конверт с 10 рублями. Вы рассуждаете так:

    Либо во втором конверте 5 рублей, либо 20. Поскольку у меня нет никакой дополнительной информации, вероятности этих событий равны, и поскольку возможный выигрыш больше возможного проигрыша, я буду менять конверт. Я правильно вас понял?

    Такая логика допустима, если количество денег во втором конверте определяется уже после выбора вами первого случайным образом между значениями 5 и 20.

    В данном случае же содержимое второго конверта определено заранее, и соответственно не является случайной величиной. К количеству денег во втором конверте вообще не может быть применено понятие вероятности. Поэтому построение ожидаемой суммы выигрыша некорректно само по себе.

    ОтветитьУдалить
  22. bkonst:
    да, читал, и выше я уже признал их и свою ошибку

    ОтветитьУдалить
  23. Анонимный28.08.2009, 18:20

    Илья, ваше убегание от сложной "теоретической" к несформулированной "реальной" задаче неспортивно. Налицо подмена понятий.

    ОтветитьУдалить
  24. Необходимо еще пояснить, что выбор использования субъективных вероятностей и изменения исходов зависит от условий задачи.


    Если у нас четко оговорено, что вероятность 1/2, то работает изменение утилит в зависимости от того, что мы рискуем потерять.

    Если вероятность того, что во втором конверте сумма вдвое больше неизвестна, то есть существует класс распределений P(2x)=q, где q из [0,1], то следует использовать условные вероятности, задавая q как склонность к риску в принятии данного решения. Очевидно, что склонность эта будет зависеть во многом от того чем Вы рискуете, и таким образом все опять сводится к первому случаю.

    ОтветитьУдалить
  25. Илья! Я долго размышлял над задачей, статьями и комментариями по мере их появления. Каковы итоги обсуждения? Попробую своими словами, как понял. Не буду касаться вопросов образования. Здесь Вы сказали много, и я с Вами полностью согласен. Ограничусь парадоксом. Поправите, где не так.

    1) Нет оснований полагать, что денежные суммы С/2 и 2С могут находиться во втором конверте с равными вероятностями.
    Это прозвучало довольно четко, хотя Ваши объяснения не показались мне корректными. Вы рассматривали частные случаи, в которых меняли условие задачи: считали выигрыш известным или ограниченным известным числом. По сути Вы показали, что если есть информация о распределении денег по конвертам, то вероятности сумм С/2 и 2С не могут быть одинаковыми. Однако я с этим выводом согласен по соображениям, которые приведу ниже.

    2) Средний выигрыш при взятии второго конверта невозможно рассчитать, так как неизвестны вероятности сумм С/2 и 2С (неопределенность).
    Это следует из предыдущего пункта.

    Далее о том, что я не понял.

    Не очень понятно с симметрией: знание суммы С в первом конверте нарушает ее или нет? Я согласен с 7vies, что знание суммы С не нарушает симметрии. Следовательно, средняя сумма в первом и втором конвертах должна быть одинаковой.

    Не очень понятно и со смешанными стратегиями, то есть с выбором конверта в зависимости от суммы С. Сказано, что она дает более высокий средний выигрыш, чем выбор всегда второго конверта (или первого). Объяснения нет, сказано, что это естественно. Надо полагать и в том случае, когда информация о распределении денег по конвертам отсутствует. Впрочем смешанная стратегия не обязательна при обсуждении парадокса.

    Теперь немного своих рассуждений.

    Итак, в одном конверте А, а в другом 2А денег. Первым считаем первый выбранный случайным образом конверт, а оставшийся - вторым. В первом конверте X, а во втором Y денег.
    Все величины А,X и Y - случайные, но X и Y тесно связаны между собой.

    Рассмотрим чистые стратегии. Тогда информация о содержимом первого конверта не используется по определению.
    Первая стратегия - выбирать всегда первый конверт.
    Учитывая случайный выбор первого конверта величина Х с равными вероятностями принимает два значения: А и 2А. Средний выигрыш составит 1,5А.
    Вторая стратегия - всегда менять первый на второй.
    При Х=А величина Y=2А, а при Х=2А, Y=А. Т.о. Y принимает такие же значения А и 2А тоже с равными вероятностями. Средний выигрыш равен тоже 1,5А.
    Равенство средних выигрышей
    подтверждает интуицию, но не объясняет парадокса.

    Учитывая связь между величинами X и Y сложно одну из них измерять другой. Удава можно измерить в попугаях или мартышках, но не его половинами или четвертями. При взгляде со стороны, во втором конверте могут быть суммы А или 2А, которые отличаются в 2 раза. А при рассуждениях через величину Х - суммы Х/2 или 2Х, которые отличаются в 4 раза. Это настораживает. Я не отвергаю категорически рассуждения через Х, однако их надо обосновать и правильно применять.

    Когда мы обнаружили в первом конверте С денег (Х=С), то получили информацию о величине А. Раньше ее распределение было неизвестно. Теперь известно, что А принимает одно из двух значений: С и С/2 с некоторыми вероятностями p1 и p2 соответственно. Вероятности зависят от закона распределения величины А, и нет никаких оснований считать их равными.
    С вероятностью p1 величины принимают следующие значения: А=Х=С, Y=2C=2А; а с вероятностью p2 - значения А=С/2, Х=С=2А, Y=С/2=А.
    Среднее значение выигрыша во втором конверте равно 2Сp1 + (C/2)p2.

    В общем случае вероятности p1 и p2 будут меняться в зависимости от С.
    А может я и ошибаюсь.

    ОтветитьУдалить
  26. Анонимный28.08.2009, 18:30

    Предлагаю следующую модель внесения денег в конверты: в первый кладем действительное а из интервала (0;+inf), во второй 2а. С вероятностью 50% конверты перенумеровываем.
    По-моему тогда обсуждаемое отношение вероятностей будет 50/50.

    ОтветитьУдалить
  27. Всех приветствую. Хочу поблагодарить автора за эту и другие статьи.
    Я считаю, что подобного рода задачи рассматриваются в "теории принятия решений". Называются они "задачами принятия решений в условиях полной неопределённости", и понятно почему: мы не можем считать, что во втором конверте 2*Х и Х/2 могут оказаться с равной вероятностью. Почему мы не можем так считать - это не оговорено в условиях задачи.
    Тот метод, который уравнивает вероятности, называется "принцип недостаточного основания" (что-то есть в этом названии =)). Он, естественно, вводит дополнительные предположения, которые в самой задаче не предполагаются.

    ОтветитьУдалить
  28. Уважаемый аноним, предлагающий следующую модель внесения денег в конверты: «в первый кладем действительное а из интервала (0;+inf), во второй 2а».

    Вы только что сформулировали очень важный вопрос для этой задачи! Математика не знает равномерного распределения чисел на бесконечном интервале.

    Мы умеем сформулировать, что такое случайное число от A до B, если A и B конечные. Но для бесконечных значений понятие случайно величины не определено в современной математике. Поэтому предложенный Вами сценарий невозможно реализовать.

    ОтветитьУдалить
  29. Уважаемый Илья,
    боюсь заблуждаться, но мне кажется распределения на бесконечных интервалах всё же используются в теории вероятностей в расширенной аксиоматике. Вот, например, на википедии http://ru.wikipedia.org/wiki/%D0%90%D0%BA%D1%81%D0%B8%D0%BE%D0%BC%D0%B0%D1%82%D0%B8%D0%BA%D0%B0_%D0%9A%D0%BE%D0%BB%D0%BC%D0%BE%D0%B3%D0%BE%D1%80%D0%BE%D0%B2%D0%B0

    ОтветитьУдалить
  30. Весь "пародокс" в том, что невозможно равновероятно распределить все целые числа.

    А значит условие равнозначности конвертов надо принять за характеристику распределения общего капитала игры.

    Получаем 2хр+0.5х(1-р)=х => p=1/3 - шанс что сумма увеличится.

    ОтветитьУдалить
  31. распределения на бесконечных интервалах всё же используются в теории вероятностей

    Они используются, но не равномерные.
    Допустим что оно есть.

    Тогда вероятность выпадения любого числа числа кроме бесконечности = 0.
    Именно ноль, а не стремиться к нему.

    Значит вероятность выпадания бесконечности 100%.

    А вот при х=(бесконечности) как раз все в порядке
    (2x+0.5x)*0.5 = x
    :)

    ОтветитьУдалить
  32. Artem, спасибо за вопрос!

    Да, распределения на бесконечных интервалах используются в теории вероятности в расширенной аксиоматике, но в данном случае они не применимы. Дело в том, что аксиома непрерывности (если мы всё ещё говорим о Колмогоровской аксиоматике) оперирует с бесконечной убывающей последовательностью событий (что вполне чётко написано по приведённой Вами ссылке на википедию). Мы же говорили про равномерную случайную величину на интервале от 0 до бесконечности - этого просто не бывает.


    Теперь отвечу на вопрос о том, почему я сужаю задачу, добавляя новые условия (предполагая то или иное распределение, выбранное Якубовичем).

    Сначала для ясности предлагаю аналогию: если у нас есть теорема про про все возможные треугольники, то вправе ли я применять её к равносторонним? Да, конечно. Даже если в теореме не было сказано, что треугольник может быть равносторонним. Теорема ведь обо всех!

    Аналогично с этой задачей: если утверждается, что стратегия "меняй конверт" всегда выгодна, то она должна быть выгодной в конкретных случаях. Поэтому разговоры о том, что я убегаю в частные случаи, здесь неуместны.

    Представьте, что кто-то утверждает, что у всех треугольников хотя бы один угол составляет 30 градусов. Можно долго искать ошибку в его доказательстве, чтобы объяснить ему, почему он неправ. А можно показать ему равносторонний треугольник - у него нет углов в 30 градусов, что полностью опровергает теорему. А раз она не работает в частном случае, то не может называться общей теоремой.

    Желающим разобраться в проблеме рекомендую статью википедии - там всё неплохо разложено по полочкам (в русской википедии, увы, взят материал из обсуждаемой статьи, поэтому в ней пока нет правильных рассуждений). Успехов в освоении теории вероятности!

    ОтветитьУдалить
  33. Илья и Антон, спасибо за пояснения, теперь я понял что вы имели ввиду и где я был не прав.
    Но я всё-таки хочу заметить, что данная задача не имеет прямого отношения к теории вероятностей (хотя, конечно, такое деление очень условно: в науке есть только проблемы и способы их решения, никаких разделов), но относится к теории принятия решений. Тут просто нет вероятностей.

    ОтветитьУдалить
  34. Как я понял из комментов, обсуждение задачи выбилось из "математической" колеи... Немного пописав программу, моделирующую эту (про 2 конверта) задачку, могу сказать, что все упирается в человека)))))

    Сейчас есть 2 подхода к решению:
    1. Математический (абстрактный). Описан в статье на мембране - есть равновероятные события получить меньшую или большую сумму. Эту задачу можно рассмотреть по другому, абстрагировавшись от денег. Например - игра в "Кто будет играть белыми в шахматы" - игрок берет конверты, в которых записаны _числа_ - а они то могут быть произвольно большими. Тогда работает логика, описанная в мембране? Работает (с точностью до "всегда").
    2. Одесский ("И гиде меня кидают?"). В этом случае рассматриваются дополнительные критерии. Ведь ограничение на сумму накладывает только то, что в конвертах деньги. Ибо деньги есть конечная величина (ну... ограниченная ВВП Земли, например). Вот тогда мы уже принимаем во внимание тот факт, что есть некоторое N и т.д....

    Есть пример из физики. Возьмем математический маятник и попробуем его показать детям. Возможно ли это? Возможно, если контролировать угол отклонения от вертикали (чтоб в дифуре колебаний sin(alpha)~ alpha ) + контролировать длину подвеса. Но как бы мы не старались, в конце концов демонстрировать мы будем именно физический маятник. Есть трение подвеса, трение воздуха, неидеальность приближения синуса угла к углу.... Решаемо ли это? Решаемо, но дааалеко не в рамках школьной математики/физики.

    Каков вывод? Нужно разделять математические и реальные задачи. Например парадокс с лошадьми можно было решать старым способом опровержения ("если есть верное утверждение противоречащее теореме, то теорема неверна"): "Может ли быть лошадь белого цвета? Может. Может ли быть лошадь черного? Может. Значит утверждение что все лошади одного цвета неверно" (ну... или как-то так:))

    В общем искать подвохи надо там, где они могут быть и быть всегда настороже. Про динозавров - опять таки, мы _знаем_, что они вымерли, те эта задача с "предусловием". А вот если спросить простого прохожего про вид насекомого, да еще и на латыни? Каков тогда ответ будет?

    ОтветитьУдалить
  35. Sergey, есть древняя легенда про математика, который смог доказать очень сложную теорему для объектов очень специфической аксиоматики - за это он получил степень кандидата наук. А потом он смог доказать, что в данной аксиоматике не существует объектов, удовлетворяющих условию теоремы (потому она и была верна) - за это он получил докторскую степень :)

    С данной задачей всё аналогично: рассуждения статьи применимы для равномерного распределения бесконечной случайной величины. Но такой величины не бывает! Поэтому всё и ломается. Я специально приводил примеры неравномерных и конечных распределений, чтобы продемонстрировать, что выкладки неверны для общего случая (вообще говоря, для возможного случая).

    Ещё раз рекомендую всем, желающим разобраться: если остались сомнения, то прочитайте, пожалуйста, статью википедии - там достаточно подробно описаны ошибки.

    ОтветитьУдалить
  36. Пардон за дубляж информации в моем комменте - статья висела достаточно долго открытой и я не видел почти половины комментариев((((

    PS. Спасибо за блог. Про математика забавно было узнать, а вот про не существование равномерно распределенной величины на бесконечном множестве - это было открытие (может забытой информации) для меня сегодня... Век живи....

    ОтветитьУдалить
  37. Анонимный28.08.2009, 22:10

    Попахивает котом Шредингера ;)

    ОтветитьУдалить
  38. Анонимный29.08.2009, 00:43

    Очень хорошая статья, я и сам не до конца понимал про конверты, пока не прочитал, спасибо за разъяснения. Только прошу, поменяйте в тексте терию вероятности на теорию вероятностей, она так правильно называется.

    ОтветитьУдалить
  39. Анонимный29.08.2009, 02:23

    Дочитал до начала 2009 года и захотелось узнать мнение о свежей книжке для подростков Астровитянка (http://lib.rus.ec/b/163295).

    Возникли ассоциации и с названием блога, и с общей тематикой записей.

    ОтветитьУдалить
  40. Анонимный29.08.2009, 02:24

    Добавлю, чтоб не было вопроса про корректность чтения с lib.rus.ec: автор сказал, что за любое распространение своей книги, в том числе и в интернете.

    ОтветитьУдалить
  41. Уважаемый аноним, спасибо, что указали на опечатку с теорией вероятностей! Раньше я об этой тонкости в именовании не задумывался (видимо, сказывалось название книги "Теория невероятности" Анчарова).

    Jerom, спасибо за ссылку, я прочитаю "Астровитянку".

    ОтветитьУдалить
  42. Хочу вернуться вот к этой фразе в заметке:
    «Главный смысл, который не все извлекают из этого анекдота, состоит в следующем: если у дающего ответ недостаточно информации, то это не значит, что разумно будет считать все возможные варианты равновероятными.»
    Так вот, она некорректна. Если каких-либо ограничений в задаче нет, то полагается считать все возможные варианты равновероятными. У нас здесь можно увидеть одно случайное событие — помещение суммы x в один конверт, и x*2 в другой. Про него ничего не сказано, поэтому нужно считать все возможные исходы равновероятными, а это, как уже писали, невозможно для бесконечного множества. Дальнейшие рассуждения в решении используют этот факт бесконечности, и потому неверны. Поэтому ошибка находится не в процессе рассуждений, а в самой постановке — задачу просто нельзя начинать решать с такими начальными условиями, нужно ограничить, например, верхнюю границу выбора суммы. Тогда фраза «при этом точно известно, что в одном из них денег в два раза больше, чем в другом» будет иметь смысл, и будет что решать дальше. Распределение вероятностей между вариантами при этом можно считать равномерным (даже если на самом деле это не так, отсутствие информации не оставляет нам другого выбора), но утверждение, что равновероятно наличие во втором конверте суммы в два раза большей или меньшей не впишется в ограничение.
    Резюме: метод решения правильный, но решать неправильно поставленную задачу нельзя.
    И по поводу динозавров на площади: если задача дана именно в такой постановке, без указания, что такое динозавр, площадь, где всё это находится, и т. п., то правильный ответ — 1/2. Но в реальности этот вопрос содержит огромное количество условий по-умолчанию, причём они разные для каждого человека. Так что ответ будет, во-первых, субъективным, а во-вторых никто не сможет его точно определить из-за огромного количества неявных условий с расплывчатыми числовыми характеристиками.

    ОтветитьУдалить
  43. Xavier, спасибо за ответ, в нём много понимания.

    Однако я хочу возразить по Вашей позиции "если что-то неизвестно, то полагается считать все возможные варианты равновероятными". Кем полагается? Есть хоть один ВУЗовский учебник, который предписывает так делать? Скорее - наоборот, серьёзные учебники вводят чёткие определения, под которые такая "вероятность" не подпадает.

    Я согласен, что может возникнуть сильный соблазн что-то такое думать (это естественное желание разобраться с недоопределённой задачей), но это всё не будет относиться к науке математике и реальному миру, потому что ожидать равновероятности от событий, о которых мы ничего не знаем, ненаучно и наивно.

    Есть сложившиеся представления математиков о теории вероятностей. И если Вы хотите ввести что-то другое, что кажется разумным, то называйте это не вероятностью, а каким-то другим словом. Тогда мы не будем путаться в терминах.

    В любом случае, спасибо за Ваш комментарий, поскольку он очень содержателен.

    ОтветитьУдалить
  44. Тер. вер. работает только если известна природа (распределение) случайных величин, либо она не важна.
    Это случай сферической монетки в вакууме.

    Когда природа случайных величин неизвестна, появляется статистика.
    Она на основе множества экспериментов с одинаковыми или похожими условиями позволяет сделать гипотезу о природе случайной величины. И результат статистического исследования для непрерывной величины звучит так: Есть гипотеза, что величина имеет "вот такой" характер. И, если гипотеза верна, следующий эксперимент даст с вероятностью 99.9% значение с отрезка такого-то. Прочем чем вероятность больше, тем больше и отрезок.

    И если меняется какой-то внешний фактор, который был одинаковым все эксперименты, прогноз не сбывается совсем вообще. Это списывается на тот 0.1%.

    Даже случай неидеальной монетки строится на гипотезе, что на результат ничего не влияет.
    А что если влияет? Вдруг есть кнопка которая заставляет монетку выпадать орлом? Здесь обычно приводится отмазка, что если ее не нажали 10000 экспериментов, то вероятность того что ее нажмут в следующем не более 1/10001 но это неправда, гипотеза, что кнопка нажимается случайно.

    То есть вся теория вероятностей (и даже статистика) работает с какого-то умозрительного "пинка" о природе случайной величины.

    Так вот в этой задаче самый достоверный "пинок" это равнозначность конвертов. Из него надо делать выводы о характере распределения капитала эксперимента, а не наоборот.

    То есть ответ звучит так:
    Чтобы игра была честной, и конверты оставались равнозначными даже после вскрытия одного из них, то распределение бюджета должно быть...(плотность или функция распределения). При этом ожидание выигрыша с одного эксперимента ...(цифра!). Если-же наше распределение будет другим, то станет возможным формировать стратегии с ожиданием выигрыша более (цифра).
    Дальше можно разбирать конкретные распределения и стратегии оптимальные для них, сравнивать ожидаемый доход итд.

    ОтветитьУдалить
  45. Илья, к сожалению вы тут ошибаетесь. На мембране не обманывают, вероятность того, что во втором конверте 2С денег равна 50% (как и вероятность С/2). И не нужно тут вводить никаких ограничений. Но и кое-что там всё-таки не договаривают, видимо, чтобы подогреть интерес :)
    Есть в этой задачке одно "неявное" условие: выбор делается 2 раза!

    Если во втором конверте 2С денег, то С - меньшая из двух сумм денег (и наоборот). Уже виден подвох. Мы всегда будем выигрывать с меньших денег, а проигрывать с больших. Прибыль компенсируется убытками.

    Теперь математика, вероятности. Для наглядности частный случай: если в конверте 10 рублей, то во втором 5 либо 20. Тут нужно рассматривать две возможные начальные пары денег в конвертах: 5 и 10 или 10 и 20. Да, если в конверте 10, то с вероятностью 50% во втором 5 и с той же вероятностью 20 рублей. Как и говорит мембрана, "статистически ожидаемая" прибыль будет 0,5 х (5 - 10) + 0,5 х (20 - 10) = 2,5. Но если в первом конверте 5 или 20 рублей, то во втором 10. "Средневзвешенная" прибыль тут будет 0,5 х (10 - 5) + 0,5 х (10 - 20) = -2,5. Вот и обратная сторона медали.

    Полная формула: ((С/2 - С) + (2С - С) + (С - С/2) + (С - 2С))/4.
    Вот такая точная наука. Условия нужно формировать полностью, только тогда будет правильный ответ.

    ЗЫ: Да, просто так всегда менять конверты смысла действительно нет :)
    ЗЫЫ: На мембране в конвертах оказывается доллары... может в этом их проблема? ;)

    ОтветитьУдалить
  46. D-nV, похоже, Вы не разобрались, поэтому Вам кажется, что «вероятность того, что во втором конверте 2С денег равна 50% (как и вероятность С/2)». Я понимаю, почему интуиция подсказыает, что это верно. Но с точки зрения математика, это ошибочное представление.

    Проблема в следующем: это утверждение должно быть верным для всех случаев (обратного ведь не оговорено). А я в данной заметке специально привёл несколько частных случаев, для которых это очевидно неверное утверждение.

    В комментариях выше я уже напоминал, что если общее утверждение не является верным для частных случаев, то его нельзя называть верным.

    Ваши дальнейшие выкладки про то, что в зависимости от того, какой конверт мы возьмём первым (с большей суммой или с меньшей), мы получим +2.5 или -2.5, поменяв его на второй, корректны.


    Anton, Вы правы, что рассуждать о вероятности в данной задаче можно было бы, если бы мы знали распределение денег по конвертам. А поскольку этих данных нет, то все красивые выкладки о том, что тактика "всегда меняй конверт" выгодна - это просто фокус и трюк (чтобы не сказать "обман"). Речь в заметке идёт исключительно об этом.

    ОтветитьУдалить
  47. Нет такого распределения чтобы вероятность получит 0.5С и 2С была одинаковой для всех С.

    Или на обычном языке. Во всей вселенной нет и быть не может объекта (живого и мертвого), который может разложить деньги по конвертам, так чтобы стратегия "всегда выбирай второй" оказалась выигрышной.

    Она выигрышная только если второй конверт формируют после вскрытия первого.

    ОтветитьУдалить
  48. «Кем полагается? Есть хоть один ВУЗовский учебник, который предписывает так делать?»
    Это придумал не я, и даже не мой преподаватель по терверу. Не вспомню сейчас, откуда это бралось, но это предположение вполне законно. Вы и сами им пользовались в заметке про парадокс Монти Холла; иначе почему для трёх шкатулок вероятность нахождения приза в каждой равна 1/3? А вдруг одна из них больше или меньше других, или вообще с прозрачной крышкой и её содержимое видно. Именно потому, что ни об одной шкатулке нет дополнительной информации, они и считаются равноправными, а нахождение в них приза равновероятным.
    Особенность задачи с конвертами в том, что нам неявно подсовывается невозможный объект — равномерное распределение по бесконечному множеству.
    И ещё раз. Решение ошибочно не потому, что «стало быть, в другом конверте лежат либо $5, либо $20 с вероятностью 50 х 50», а потому, что задачу вообще нельзя начинать решать в такой постановке. Точно так же, как нельзя решать задачу о самолёте на ленте транспортёра, или, например, такую:
    Пусть гипотенуза прямоугольного треугольника равна 10 см. Найти его площадь, если высота, опущенная на гипотенузу равна 2 см.
    Дальше я, возможно, скажу глупость, но мне представляется, что всё так и есть.
    Чтобы строго применить теорию вероятности к какой-либо случайной величине, нам нужно знать для неё закон распределения. Но в реальности мы этот закон можем никогда и не узнать, потому его приходится угадывать с той или иной надёжностью. Anton это уже написал. Но для надёжного определения закона может не хватать опытных данных. Тогда для объектов реального мира можно попытаться применить теорему Чебышева и считать распределение нормальным. Когда же данных не хватает даже на то, чтобы с достоверностью определить параметры этого нормального распределения, горб всё больше расплывается и превращается в прямую, что на ограниченном отрезке или на конечном числе вариантов даёт равномерное распределение. То есть при отсутствии информации равномерное распределение — наилучшее предположение из возможных. Это всё, конечно, лирика, правдоподобные предположения, но иногда кроме этой лирики ничего и нет; есть целые разделы математики, построенные на разумных предположениях.
    Теперь про то, что утверждение должно быть верным для всех случаев.
    При работе с веорятностями грань между частным случаем и изменением исходных данных очень тонка. Приведённые «представьте себе» дают новые задачи, их даже решать можно.

    ОтветитьУдалить
  49. Xavier, в парадоксе Монти-Холла предполагается, что шкатулки равнозначные, а мы берём случайную из них (известно, что никто правила не нарушает, никто никому не подсуживает, одну из трёх шкатулок мы выбираем случайно). Поэтому мы можем утверждать, что вероятность нахождения приза в случайной выбранной шкатулку равна 1/3.

    А в данной задаче никаких предположений о распределении неизвестно. Поэтому некорректно их выдумывать, чтобы что-то обосновать.


    Парадокс двух конвертов вызывает интерес не из-за того, что нам подсовывают несуществующий объект. А из-за того, что мы привычно додумываем равновероятность там, где нам о ней достоверно неизвестно (и где её, вообще говоря, нет). И получается, что мы сами себе выдумали несуществующий объект, а потом доказали для него "интересное" утверждение (о том, что всегда выгодно менять конверты).

    Дело не в том, что задачу нельзя решать в такой постановке, а в том, что её нельзя решать таким образом (привлекая некорректные утверждения).

    Выше уже подробно расписали, что если выбирать случайный конверт (или противоположный случайному - при тактике "бери другой") и всё посчитать честно, то средний выигрыш на каждой итерации будет 1.5A (если в конвертах A и 2A). И это вполне логично и правильно.

    Другими словами, если задачу решать корректно, то не возникает парадоксов, всё согласуется с жизненным опытом. А если решать некорректно, то можно получить любой результат (в том числе, совершенно обескураживающий).

    ОтветитьУдалить
  50. Хорошо. Вот часть формулировки парадокса Монти Холла: «Suppose you're on a game show and you're given the choice of three doors. Behind one door is a car; behind the others, goats. The car and the goats were placed randomly behind the doors before the show. The rules of the game show are as follows: After you have chosen a door, the door remains closed for the time being». Где здесь сказано о способе распределения приза? Кто сказал, что авто не ставят чаще за первой и второй дверью, чем за третьей, просто потому, что рабочим ближе её вести? Даже если на самом деле авто оказывается в половине случаев за первой дверью, в четверти за второй и в четверти за третьей, это никак не влияет на решение, потому что задача решается отностительно игрока, который не знает этого. А раз он не знает, ему остаётся брать лучшую из возможных для него оценок.
    В задаче о конвертах два варианта решения: либо нужно построить максимально согласующуюся с неявными условиями модель, и считать для этой модели вероятность, тогда получится решение по имеющемуся набору данных. Ограничений тут может быть много: либо существование определённой максимальной суммы, либо примерный средний выигрыш, либо какая-нибудь статистика. Все эти ограничения приходят извне и к математике, собственно, не относятся.
    Второй вариант — объявить, что данных для решения недостаточно, а предположения по-умолчанию приводят к некорректной формулировке, и прекратить все попытки решения.
    Поэтому я не понимаю, как можно задачу решать корректно, и при этом не строить предположений. Предположения неизбежны.

    ОтветитьУдалить
  51. Xavier, заметьте, что разбирая парадокс Монти-Холла, мы не используем равномерность распределения призов. Первым ходом человек выбирает случайную дверь, потому что у него нет никаких сведений о том, где приз. Если приз за одной из дверей, а дверь выбираем одну из трёх, то с вероятностью 1/3 мы попадём в приз (мы ничего не выдумываем - всё чисто, без лишних некорректных предположений).

    Вы пишите, что в задаче с конвертами есть два варианта решения: 1) догадаться, какая модель соответствует условию - тогда всё вычислить, 2) сознаться, что данных недостаточно - тогда прекратить решать.

    Но мы-то разбираем логический парадокс! Кто-то чего-то некорректно домыслил, пойдя по первому пути - получилась ерунда, которая, как минимум, не согласуется с эксперементами (уже это должно навести на мысль, что есть ошибка в теоретических выкладках). И я пытаюсь показать, почему возник парадокс - говорю, что имело места фантазия, поэтому удалось "доказать" то, чего нет в реальности.

    А Вы, если я правильно понимаю, доказываете, что эта фантазия была необходима, потому что данных в условии недостаточно. Если данных недостаточно, то не надо делать выводы, как будто данных хватает! Иначе будут получаться какие угодно парадоксы.


    По второму пункту: Вы пишите, что не понимаете «как можно задачу решать корректно, и при этом не строить предположений». Вопрос в том, что значит "решать задачу". Обратите внимание на комментарий Chester'а - всё честно расписано без выдумывания равномерности распределения.

    Некоторые величины можно вычислить, имея недостаточно данных. А то, что вычислить нельзя, то вычислить нельзя (в строгих дисциплинах). Если бы мы занимались неточной наукой, где постоянно делаются оценки "на глазок", то додумывание неизвестного было бы корректным. Но теория вероятностей - точная наука. И если её некорректно применять, то возникают парадоксы.

    ОтветитьУдалить
  52. «Если приз за одной из дверей, а дверь выбираем одну из трёх, то с вероятностью 1/3 мы попадём в приз (мы ничего не выдумываем - всё чисто, без лишних некорректных предположений)». В том и дело, что с предположениями. Если ведущий скажет перед выбором, что в половине случаев приз за средней дверью, то выбрав среднюю дверь мы (в этих условиях) получаем приз с шансом 1/2. Если ведущий этого не скажет, то с шансом 1/3. Реальное распределение при этом не меняется, меняются лишь наши знания о нём.
    «Но мы-то разбираем логический парадокс!» Если я ничего не путаю, это не парадокс, а софизм.
    «Кто-то чего-то некорректно домыслил». Вот именно, что некорректно домыслил. Всё, кроме этого предположения сделано верно. В том числе и вывод о равенстве вероятностей. Ошибка в том, что не проверено «предположение по-умолчанию», а такие проверки должны делаться до начала доказательства.
    «Если данных недостаточно, то не надо делать выводы, как будто данных хватает!» Да, но это не всегда повод отказаться от решения. Во всяком случае тогда, когда задача взята из реального мира, а не построена из искусственных объектов. В реальных задачах приходится так или иначе трактовать условия, вводить ограничения, и т. п. Разумеется, все введённые поправки при этому нужно указывать в решении. Наука-то точная, но при столкновении со «здравым смыслом» и другими субъективными вещами, она может оказаться бесполезной.
    Потому правильный ответ либо «недостаточно данных» (глядя на условие и не порываясь что-либо доказывать), либо «пусть максимальная сумма выигрыша ограничена величиной A, тогда…»

    ОтветитьУдалить
  53. Xavier, наконец-то мы друг друга поняли, мне кажется.

    Одна тонкость: Вы пишете «Если ведущий скажет перед выбором, что в половине случаев приз за средней дверью...», как будто забывая, что это противоречит условию задачи. С тем же успехом можно было предполагать, что ведущий схитрит, вообще не поместив приз. Однако мы должны строго следовать условию задачи: приз находится за одной из дверей, а участник выбирает случайную из них, потому что не имеет других сведений - это даёт вероятность выбора приза 1/3.

    Про отличие реального мира и теории я с Вами согласен полностью, важно только не смешивать решение теоретической задачи и практические особенности реального мира.

    Xavier, спасибо большое за Ваши содержательные комментарии, я уверен, они многим будут полезны!

    ОтветитьУдалить
  54. Да, думаю, поняли. Да и с самого начала говорили примерно об одно, только разными словами.
    Условия Монти Холла я поменял только для того, чтобы показать, что важен не сам закон распределения, а наше знание о нём. А полное отсутствие знаний обычно выражается равномерным распределением.
    Совсем не смешивать практику с теорией, к сожалению, не получается, иначе теряется смысл, но понимать, что же мы сейчас решаем, и откуда получено то или иное знание, нужно.
    Вам тоже спасибо, приятно почитать хоть что-то определённое после той магической математики, которой сейчас занимаюсь.

    ОтветитьУдалить
  55. С парадоксом Монти-Холла особых сложностей не вижу. Это матричная игра ведущего с игроком. У каждого из них своя стратегия. Если игрок не знает стратегию ведущего, он выбирает двери с равными вероятностями 1/3. То есть он применяет смешанную стратегию, обеспечивающую ему выигрыш с вероятностью 1/3. И это не зависит от стратегии ведущего. Если же ведущий обнародует свою стратегию, у игрока появляется возможность изменить свою и добиться большего выигрыша.
    Если ведущий ставит приз с вероятностью 1/2 за средней дверью, а игрок применяет все ту же стратегию - выбирает двери с равными вероятностями, то он по-прежнему получает выигрыш с вероятностью 1/3 независимо от того, рассказал ведущий свою стратегию или нет. Важно не просто знать стратегию ведущего, а реагировать на нее.
    "Если ведущий скажет перед выбором, что в половине случаев приз - за средней дверью, то выбрав среднюю дверь мы получаем приз с шансом 1/2. Если не скажет, то с шансом 1/3. Реальное распределение при этом не меняется, меняются лишь наши знания о нем." Но вероятность 1/2 или 1/3 появляются не в результате наших знаний, а в результате наших действий. Какую стратегию применяет игрок? Если все ту же, то независимо от знаний вероятность выигрыша равна 1/3. Ведь среднюю дверь мы выбираем не всегда, а с вероятностью 1/3.
    Если же игрок прореагировал на информацию, изменил стратегию и всегда выбирает среднюю дверь, то вероятность выигрыша становится 1/2.

    ОтветитьУдалить
  56. Xavier, я настаиваю, что ошибка, приводящая к парадоксу именно "50/50 что во втором конверте 0.5С или 2С". Единственная во всем рассуждении, приводящая к очевидно ложным выводам.
    А значит парадокс доказывает его ложность.

    50/50 применимо только к равнозначным объектам.
    Конверты равнозначны, для человека ничего о них не знающего, до того пока они не вскрыты. Если играем первый раз, то и после вскрытия одного они остаются одинаковыми, по тому что мы ничего не знаем о распределении суммы туда засунутой. Капиталы 3С и 1.5С остаются из всех возможных, но не являются равновероятными. Почему - писал выше.

    С дверьми все по другому. Они равнозначны для игрока. У ведущего нет стратегии, его поведение оговорено задачей - он бот.
    Сменить выбор там - значит открыть две двери вместо одной. Именно поэтому шансы выбранной сначала двери 1/3, а оставшейся после вскрытия ведущим 2/3. Если второй игрок войдет в комнату когда двери уже две, для него они не будут различаться (1/2 1/2). Когда он получит информацию от первого игрока - двери перестанут быть одинаковыми (1/3 2/3). А если он заглянет хотя-бы за одну из них вероятности станут 1 и 0.

    ОтветитьУдалить
  57. То есть вся соль парадокса: Нельзя считать исходы равновероятными если исходов бесконечно много.

    А парадокс это утверждение доказывает для частного случая с множеством исходов 1, 2, ...

    ОтветитьУдалить
  58. Да нет же. Это фраза не может быть главной ошибкой, потому что некорректные фразы уже были до неё. Например вот эта: «Допустим, вы увидели $10». Эта фраза подразумевает, что мы провели эксперимент, получили конверт, и т. д. Она должна тут же породить вопрос: а как мы, собственно, провели эксперимент, из чего выбрали, какие предположения использовали, и т. д. Сама попытка о чём-то рассуждать до выяснения того, при каких условиях существует объект, некорректна. Точно так же, как в задаче о площади треугольника, которую я недавно приводил.

    ОтветитьУдалить
  59. Xavier, я про задачу по сути, а не про тот маразм написанный на мембране.

    На всякий случай откину шелуху

    Постановка: Два конверта в одном денег в два раза больше чем в другом. Нам разрешают открыть любой конверт, посмотреть сколько там денег и поменять выбор.

    Стратегия (меняй конверт): ...
    Обоснование:
    гипотеза 1: Все капиталы равнозначны.
    Пусть во вскрытом С денег, тогда в другом либо 0.5С (исход 1) либо 2С (исход 2). По (гипотезе 1) оба исхода равновероятны. Тогда мат ожидание дохода стратегии 0.5(0.5С + 2С) = 1.25С > C

    Но стратегия бредовая, а значит гипотеза 1 не верна.

    ОтветитьУдалить
  60. «Но стратегия бредовая» — потрясающая математика. :-) Но в целом в такой формулировке не возражаю.

    ОтветитьУдалить
  61. Илья, Anton. Вы всё-таки ошибаетесь, вам нужно внимательнее изучить основы курса теории вероятностей и принять такие математические понятия, как бесконечность.

    По условию задачи, в конверте может быть либо С, либо 2С денег. Как при вытаскивании первого конверта, так и при его замене имеем равновероятные события получить С или 2С денег. "Равновероятные" значит, что вероятность каждого равна 50%. Т.е. она стремится к 50% при количестве событий, стремящемся к бесконечности. Это основы, и странно что мы их вообще тут обсуждаем. Их не нужно подвергать сомнению. И, конечно, это не противоречит задаче и здравому смыслу, что я и доказал в своём предыдущем посте. Нужно только правильно и полностью составлять формулы, "формулировать задачу". На мембране составили только половину, отсюда ошибка. А не из-за распределения вероятностей.

    Если не верите, простой пример:
    вы же не будете отрицать, что вероятность выпадения орла (решки) при подкидывании монеты равна 50%? Но если подкинуть монету одни раз, то выпадет либо орёл, либо решка. Пусть, выпал орёл. Это частный случай, но из этого не следует, что при подкидывании монеты вероятность выпадения орла равна 100%. И он не опровергает распределение вероятностей в целом. Вы же сами писали про лошадей, и сами же пытаетесь пользоваться такими доказательствами :) Да, если добавить в задачу условие о предельной (минимальной) сумме в конвертах, то распределение вероятностей будет другим. Но если такого ограничения нет, будет 50%.

    Ещё раз. 50% не говорит о том, что всегда выгодно менять конверт. Наоборот, в случае с конвертами именно это позволяет доказать, что менять конверты бессмысленно при любых отношениях денег в них (кроме ноля).

    ОтветитьУдалить
  62. Anton
    Но стратегия бредовая, а значит гипотеза 1 не верна.
    Стратегия бредовая не потому, что гипотеза 1 не верна, а потому, что вы (т.е. изначально мембрана) неправильно составили формулу расчёта "матожидания" для случая с конвертами. Я привёл правильную (полную) формулу выше. Она показывает, что нет смысла менять конверты. Это уже не бред. И тогда вывод, соответственно, получается прямо противоположный: гипотеза 1 верна :)

    ОтветитьУдалить
  63. D-nV, Ваша "Полная формула: ((С/2 - С) + (2С - С) + (С - С/2) + (С - 2С))/4." не различает конверты. Нельзя учитывать второе "если" потому что в первом конверте не может быть "5 или 20" пока мы их различаем (тот с 10-ю у нас в руках).

    Эта формула будет справедлива если нам разрешили заглянуть в конверт, мы запомнили, что рублей там 10, но забыли в какой конверт заглядывали.

    Пример для наглядности. Пуст мы знаем, что бюджет 30. По Вашей методике:
    Заглянули в конверт 1 - там 10. Итого ожидание дохода от смены конверта 1(20-10)
    Но ведь в первом могла быть и 20-ка. 1(10-20) итого полная формула 0.5( (20-10) + (10-20) ) = 0.

    ОтветитьУдалить
  64. Anton, нет никакой разницы, сколько конкретно будет в первом конверте. Мы не знаем "общий бюджет". Я рассмотрел общий вариант, со всем возможным бесконечным множеством сумм в конвертах. С точки зрения открывшего первый конверт и увидевшего С, пары всегда две: С и 2С, С/2 и С. Эти варианты равнозначны и равновозможны (нет никаких ограничений по бюджету).

    В принципе можно не заморачиваться с двумя парами (я это рассматривал для наглядности, но видимо только больше запутал). С позиции раздающего пара всегда одна, это М (меньшая сумма) и Б (большая сумма), причём М=Б/2. Вытащить М или Б - равновероятно, как получить орёл или решку. Это с точки зрения играющего неясно, Б он вытащил или М, поэтому приходится вводить вторую возможную равновероятную пару. Тут и ошибка мембраны: вторую пару они ввели, а ожидаемую сумму посчитали как будто пара одна. Они просчитали Б=10 из одной пары и М=10 из другой. Но не включили в расчёты равновозможные М=5 из первой пары и Б=20 из второй. Вот я и расширил формулу.

    Но вообще это не нужно, если рассматривать просто М и Б из одной пары.

    ОтветитьУдалить
  65. Если рассмотреть все варианты
    s - общий бюджет раунда.
    1. Пока конверты не открыты
    /sum_{i=1}^/inf P{s=i}*0.5*(i/3 + i*2/3)
    2. Когда конверт открыт и обнаружено С. От смены конверта:
    /sum_{i=1}^/inf P{s=i|в одном из конвертов С}*0.5(i/3 + 2i/3) - C = P{s=3C}*0.5(C+2C) + P{s=1.5C}*0.5(C/2 + C) - C = C(P{s=3C}3/2 + P{s=1.5C}3/4 - 1)

    Если для всех i,j P{s=i} = P{s=j} (равновероятность)
    То после вскрытия
    P{s=3C} = P{s=1.5C}
    P{s=3C} + P{s=1.5C} = 1
    Значит
    P{s=3C} = P{s=1.5C} = 0.5

    Тогда ожидание смены конверта
    C(3/4 + 3/8 - 1) = С(6 + 3 - 8)/8 = C/8 > 0

    Где не учтены все варианты?

    PS.
    P{A} - вероятность события А
    P{A|B} - вероятность события А, если В случилось
    /sum_{нижний индекс}^{верхний индекс} - знак суммы
    /inf - знак бесконечности

    ОтветитьУдалить
  66. D-nV, я понял, где Вы нас не понимаете :)

    Вы говорите «По условию задачи, в конверте может быть либо С, либо 2С денег. Как при вытаскивании первого конверта, так и при его замене имеем равновероятные события получить С или 2С денег. "Равновероятные" значит, что вероятность каждого равна 50%.»
    (жирным я выделил ошибочную часть)

    Это утверждение совершенно верно для вытаскивания первого конверта! И с этим никто не спорит!

    Но с заменой конверта есть тонкость. Моё утверждение: если мы выбрали один конверт, а в нём оказалось x денег, то не всегда правда, что во втором будет 2x с вероятностью 50%.

    При вытаскивании первого конверта удвоенная (бОльшая из двух) сумма будет вынута с вероятностью 50%. Но когда мы посмотрим на второй, то вовсе не факт, что в нём удвоенная сумма будет с вероятностью 50% (мало того, что мы не можем доказать, что там будет 50%, ещё и есть примеры, показывающие вероятность, отличающуюся от этой).

    Пожалуйста, перечитайте эту заметку или соответствующую статью Википедии - там проблема описана детальнее и другими словами. И Вы поймёте, что эти два утверждения различаются.

    Успехов!

    ОтветитьУдалить
  67. Вы утверждаете, что При вытаскивании первого конверта удвоенная (бОльшая из двух) сумма будет вынута с вероятностью 50%. Но когда мы посмотрим на второй, то вовсе не факт, что в нём удвоенная сумма будет с вероятностью 50%. Из ваших слов как раз следует, что тактика постоянно замены конвертов будет работать. Но вы же сами согласны с тем, что это не так. Получается противоречие.
    И что значит ваша "переменная" или "неопределённая" вероятность? Она хоть больше или меньше 50%? Если больше, то менять выгодно, если меньше - не выгодно. Но мы знаем, что менять всегда нет смысла. Вероятность именно равна 50%.

    ОтветитьУдалить
  68. D-nV
    Чтобы появление во втором конверте большей и меньшей сумм было равновероятно, необходимо одновременное выполнение двух условий:
    1. Максимальная сумма не должна быть ограниченной сверху, иначе при сумме q в первом конверте большей max(s)/2 вероятность получить q*2 во втором равна нулю.
    2. Все суммы должны быть равновероятны (надеюсь, очевидно, почему).
    Однако одновременное выполнение этих условий невозможно.
    «И что значит ваша "переменная" или "неопределённая" вероятность?»
    Исходных данных недостаточно, чтобы определить вероятность. Обычное предположение равновероятности всех возможностей несостоятельно, при других предположениях результаты будут разные.

    ОтветитьУдалить
  69. Xavier
    1) Верхнего и нижнего предела нет в условии.
    2) Все суммы равновероятны, т.к. иного в условии не оговорено.
    Почему эти условия вдруг несовместимы?
    Что значит, исходных данных недостаточно? Это всё равно что говорить "вероятность выпадения орла не равняется 50% т.к. мы не знаем, прямая ли монета". С математической точки зрения это бред.

    ОтветитьУдалить
  70. D-nV, все суммы не могут быть равновероятны в принципе. Уже про это писал.

    ОтветитьУдалить
  71. Dn-v, если Вы пишете «Из ваших слов как раз следует, что тактика постоянно замены конвертов будет работать», то это надо обосновать. Ведь я утверждаю, что из моих слов следует прямо противоположное. Пожалуйста, перечитайте заметку и комментарии выше, чтобы не плодить одинаковые вопросы и одинаковые ответы.

    Если Вы что-то не понимаете, а до Вас уже задавали такой же вопрос (например о том, почему нельзя считать события равновероятными, если не сказано, что они равновероятны), то постарайтесь понят ответ. И если не поймёте ответ, то уже уточняйте, что именно в нём сложно.

    Но попробую коротко ответить:

    1) Ответ на «И что значит ваша "переменная" или "неопределённая" вероятность? Она хоть больше или меньше 50%?»:
    Не переменная, а именно неопределённая. Неизвестно, чему она равна. Если что-то неизвестно, то некорректно его с чем-то сравнивать.

    2) Ответ на «Что значит, исходных данных недостаточно? Это всё равно что говорить "вероятность выпадения орла не равняется 50% т.к. мы не знаем, прямая ли монета"»:
    Как раз про монеты всегда оговаривается, что они симметричны. А о распределении в этой задаче ничего неизвестно.

    Пожалуйста, разберитесь с ответами на Ваши вопросы, данными в заметке и комментариях выше, потому что я считаю неправильным повторять их много раз.

    ОтветитьУдалить
  72. Ваши утверждения тоже не обоснованы. "нельзя считать события равновероятными, если не сказано, что они равновероятны". В задаче сказано, что в конверте может быть либо то, либо то. И нет никаких других ограничений. Либо орёл, либо решка. Подразумевается равновероятность. Если это не очевидно, то спор бессмыслен.

    ОтветитьУдалить
  73. D-nv, я правильно понял Вашу фразу «Ваши утверждения тоже не обоснованы» как согласие с тем, что Ваши утверждения не обоснованы? Если так, то наша переписка - это не поиск истины, а пустое произнесение слов. Тогда я прекращу попытки объяснить Вам ошибки в Ваших рассуждениях.

    Если нет, то давайте вернёмся к делу: Вы сделали вывод из моих слов, который из них не следует (а следует противоположный). Я попросил Вас последовательно пояснить, как именно Вы сделали такой вывод, чтобы указать на конкретную ошибку в выкладках. Вы готовы это сделать?

    ОтветитьУдалить
  74. D-nV, обосную попроще.

    Основное свойство вероятности - сумма по всем возможным взаимоисключающим исходам равна 1.

    монета: у нее две стороны, она симметрична (идеальная)
    система:
    Р{орел} = P{решка}
    Р{орел} + P{решка} = 1
    Отсюда: Р{орел} = P{решка} = 0.5

    кубик: У него 6 граней и он симметричен.
    система:
    /sum_i=1^6 Pi = 1
    Pi = Pj для всех i,j из [1,2,...,6]
    Отсюда: Pi = 1/6 для всех i

    Равновероятный генератор натуральных чисел
    /sum_i=1^/inf Pi = 1;
    Pi = Pj для всех натуральных i,j.
    Эта система несовместна. То есть нет Pi удовлетворяющих этим условиям.

    Если не видно поясняю.
    Пусть a=Pi для всех i, они ведь равновероятны.

    Какое а>0 мы не возьмем, сумма ряда a+a+a+... расходится, равна /inf. А значит единице не равна - неподходит.
    a=0 тоже не подходит, потому что сумма ряда будет нулевой. То есть не равна единице.
    Осталось a<0, но по свойству вероятности она не подходит тоже, да сумма ряда разойдется.

    Итого перебрали все вещественные числа, то есть все возможные вероятности. Ни одно не подошло.

    Значит не могут быть все натуральные числа равновероятными.
    Можно повторить эти рассуждения для любого счетного бесконечного множества.

    Боюсь проще обьяснить не смогу.

    ОтветитьУдалить
  75. Илья
    "Ваши утверждения не обоснованы?"
    Выше я доказывал, что нет смысла всегда менять конверты, используя равновероятность получить большую или меньшую сумму в обоих случаях.
    Вы можете доказать тоже самое используя своё утверждение о "неизвестной" вероятности? Т.е. есть вероятность поменять сумму на меньшую или на большую, но вероятность, по вашим словам, не одинаковая. Как тогда доказать, что средняя прибыль в любом случае при любых условиях будет равна 0?

    Антон
    "не могут быть все натуральные числа равновероятными"
    Я могу только согласиться с тем, что в бесконечном множестве мы не можем посчитать вероятность выпадения каждого числа. Очевидно, она будет 1/inf. Т.е. можно сказать, что вероятность будет "бесконечно малой". Но равной для всех чисел.
    Вы же утверждаете, что вероятность выпадения, например, числа 1 не равна вероятности выпадения числа 2 из бесконечного множества чисел. Спрашивается, с какой стати?

    Вообще-то это мало относится к нашей задаче. У нас речь идёт про равновероятность выпадения большего либо меньшего числа из множества, состоящего только из этих двух чисел.

    В принципе, если вам мои рассуждения кажутся уж совсем бредовыми, то мы можем дальше это не обсуждать. Просто интересно понять, кто где ошибается. Я для себя понял, что Илья видит ошибку мембраны в распределении вероятностей, Антон в бесконечности возможных сумм. В итоге вы даже нашли точку соприкосновения. Я лично вижу ошибку просто в неправильном расчёте ожидания, не учитывающем половину возможных сумм при первом вытягивании конвертов, если рассматривать две возможные их пары.

    ОтветитьУдалить
  76. D-nv, благодарю за позитивный настрой. Раз он есть, то мы доберёмся до истины :)

    Итак, правильно ли я понимаю, что Вы утверждаете,
    1) раз в условии ничего не сказано про распределение, значит оно равномерное,
    2) раз в условии ничего не сказано о верхней границе, значит в конвертах могут быть суммы от 0 до бесконечности?

    Из этого Вы делаете вывод, что задачу надо решать в условии, что в конвертах находятся отличающиеся в два раза суммы, причём их распределение равномерно и неограничено.

    Если всё так, то дальнейшие выкладки не содержат ошибок - действительно, вероятности встретить x/2 и 2x во втором конверте будут равны, если в первом конверте найдено x. Кстати, это означало бы, что второй конверт всегда выгодно брать.

    Но Антон выше вполне корректно показал, что мы не можем говорить о равномерном неограниченном распределении (нет такого математического понятия). Другими словами, уточнения об условии задачи, которые Вы сделали, означают, что Вы работаете с несуществующим объектом. А для несуществующих объектов можно доказать всё (в том числе и то, что второй конверт брать всегда выгодно - что и было сделано).


    И небольшое дополнение к Вашему ответу Антону: «Я могу только согласиться с тем, что в бесконечном множестве мы не можем посчитать вероятность выпадения каждого числа. Очевидно, она будет 1/inf».
    Такая фраза показывает, что Вы не очень глубоко понимаете природу бесконечности. У математиков давно есть догововрённости, что можно делать с бесконечными объектами, а чего нельзя. Это большая и серьёзная тема - явно не для этих комментариев. Важно, что если делать некорректные вещи, то получится доказать что угодно, но это не будет иметь смысла. Поэтому, если Вы пишете 1/inf или другие некорректные вещи, то разговор выходит из плоскости математики и логики. Ещё раз повторю, что доказательство Антона несуществования равномерного неограниченного распределения корректно.

    Пишите, если остались вопросы.

    ОтветитьУдалить
  77. Илья, извините, сейчас нет возможности прочитать ваш комментарий. Я обязательно прочитаю вечером. Просто сейчас уточнил в теории вероятностей: для зависимых событий, таких, как вытягивание второго конверта, считается "условная вероятность", которая равна произведению вероятности наступления события А и вероятности наступления события Б при условии наступления события А. Т.е. Р(А,Б)=Р(А)*Р(Б/А)=Р(Б)*Р(А/Б). Вероятность вытянуть большую или меньшую сумму в первом конверте равна 50%, с этим вы согласились. Тогда вероятность вытянуть больую или меньшую сумму во втором конверте равна 0,5*1=0,5. Т.к. вероятность вытянуть большую сумму после меньшей и меньшую после большей равны 100% по условию.

    ОтветитьУдалить
  78. D-nV, отвечу последовательно:

    1) Вы пишете «Вероятность вытянуть большую или меньшую сумму в первом конверте равна 50%, с этим вы согласились». Но я с этим никогда и не спорил! Здесь вероятность именно 1/2, что я неоднократно писал выше.

    2) Вы пишете «Тогда вероятность вытянуть больую или меньшую сумму во втором конверте равна 0,5*1=0,5».

    Вы путаете разные вероятности!

    - Есть вероятность вытянуть "не первый конверт" - она равна 0.5, так как первый конверт мы вытягиваем с вероятностью 0.5.

    - А есть вероятность вытянуть конверт, в котором сумма будет в два раза больше, чем в первом. Вот её мы не можем знать, не зная распределения.

    Выше я уже писал Вам об этой путанице. Разница тонкая, но она есть. И с ней надо разобраться, чтобы перестать ходить по кругу.

    Успехов!

    ОтветитьУдалить
  79. Про бесконечно малые:
    Они используются только совместно с пределами для разрешения неопределенностей вида
    0/0 (например дифференцирование)
    0*inf (по сути интеграл)
    inf/inf

    Неопределенности нет:
    1/inf = 0
    1/+0 = +inf
    1/-0 = -inf
    1*inf = inf

    Доказательства невозможносити равновероятноси приводил для дискретного случая.
    Доказать то-же для непрерывного, советую провести Вам в качестве упражнения. :)

    Про полную вероятность. Или как получить стратегию из распределения(для дискретного случая):

    K(x) - "Капитал игры равен х"
    А(х) - "В открытом конверте х"

    P{K(3x)} = /sum_i P{A(i)}*P{K(3x)|A(i)}

    K(3x)|A(i) - "Капитал раунда 3х при сумме в открытом конверте i"

    K(3x)|A(2x) - "Проигрыш от смены конверта"
    K(3x)|A(x) - "Выигрыш от смены конверта"
    Для остальных i P{K(3x)|A(i)} = 0
    Не может быть капитал 30 баксов, если в первом конверте не 10 и не 20.

    Обозначим:
    N(x) = P{K(x)}
    V(x) = P{K(3x)|A(x)} - вероятность выиграть от смены конвертов.
    тогда P{K(3x)|A(2x)} = 1-V(2x)

    Если первый конверт выбран случайно.
    P{A(x)} = 0.5(N(3x) + N(1.5x))

    Получим:
    P{K(3x)} = P{A(x)}*P{K(3x)|A(x)} + P{A(2x)}*P{K(3x)|A(2x)} = 0.5( (N(3x) + N(1.5x))V(x) + (N(6x) + N(3x))(1-V(2x)) )

    Итого система связывающая стратегию и распределение капитала раунда:
    N(3x) + (N(6x) + N(3x))V(2x)= N(6x) + (N(3x) + N(1.5x))V(x)
    sum_i N(i) = 1
    N(i)>=0
    0<=V(i)<=1

    Спасибо за внимание
    [аплодисменты]
    [вопросы аудитории]

    ОтветитьУдалить
  80. Илья, я честно не вижу, где я путаю. По моему мнению, для нашей задачи есть почти полностью аналогичная. Подкидывая монетку, нужно набрать больше орлов. Утверждение: если всегда переворачивать монетку после подкидывания, то вероятность выпадения орла будет выше. Отличие тут только в том, что смотря на монетку мы точно можем определить, орёл на другой стороне или решка. Смотря на сумму денег, мы не можем точно сказать, бОльшая ли эта сумма или меньшая. На мембране в конце предлагается примерно следующее: считать среднюю суммы во всех полученных конвертах (или вообще придумать её наугад) и принимать решение о смене в зависимости от денег в первом конверте (если меньше, то менять). Не знаю на сколько эта тактика выигрышна, но то, что это так - похоже на правду.
    Но мы-то говорим вообще о другом. Мы меняем (или не меняем) все конверты НЕЗАВИСИМО от выпавшей в первом суммы. Это равносильно тому, что подкидывать монетку и всегда переворачивать. 50/50.

    Если всё так, то дальнейшие выкладки не содержат ошибок - действительно, вероятности встретить x/2 и 2x во втором конверте будут равны, если в первом конверте найдено x. Кстати, это означало бы, что второй конверт всегда выгодно брать.

    Да как же так? Я же выше чётко написал формулу ожидаемой прибыли при таком раскладе (50х50), и показал, что при любых условиях она равна нолю. Вы с этими расчётами согласились, а сейчас опять говорите, что менять выгоднее... :(

    Да это же даже без всяких доказательств понятно, что только при равных вероятностях будет равный результат! Что меняй, что не меняй. Что переворачивай, что не переворачивай. О чём тут можно вообще спорить?

    ОтветитьУдалить
  81. D-nV, если бы Вы на самом деле хотели разобраться, то задавали бы вопросы по уже данным Вам разъяснениям. Сейчас же складывается впечатление, что Вы плодите всё новые и новые однотипные вопросы, практически полностью игнорируя данные Вам ответы.

    Прочитайте ещё раз мой ответ, вникните в него. Сейчас Вы просто выдернули из него цитату, не анализируя её.

    Я говорил «Если всё так, то дальнейшие выкладки не содержат ошибок...» про что? Вы прочитали текст после этого?

    А там написано, что «Антон выше вполне корректно показал, что мы не можем говорить о равномерном неограниченном распределении». Применительно к Вашему цитированию моих слов, это означает, что «Не всё так».

    Поэтому не надо приписывать мне слова, с которым я не согласен. А потом разубеждать меня «в моей неправильно позиции». Я никогда не утверждал, что "всегда менять выгоднее".

    Ещё раз: если хотите разобраться, то читайте ответы, уже неоднократно данные Вам. Всё подробно расписано выше (и не один раз). И если что-то будет вызывать сложности в тех ответах, то спрашивайте. Но не надо повторять одно и то же - это показывает неуважение к отвечающим (Вы как будто не считаете нужным читать их ответы, а повторяете вновь свои старые вопросы).

    Успехов!

    ОтветитьУдалить
  82. Илья, здравствуйте - прочитал на хабре о Вашем блоге и уже второй день не могу оторваться - большое спасибо за интересные статьи, а задачки отсюда ещё и знакомым кидаю, чтоб голову поломали)

    Но вот в вопросе о конвертах я, перечитав все комментарии, всё же присоединяюсь к D-nV.

    Я считаю, что задачу без потери смысла можно переформулировать так:
    Есть 2 конверта - человек выбирает конверт. После этого, во второй ложат сумму либо меньшую, либо бОльшую.

    Я здесь не вижу каких-либо бесконечностей и невозможностей равномерного распределения - все три суммы - числа (не бесконечности), из них явно составляются 2 пары, вероятности "выпадения" которых 50%. Получается что то вроде 2 подкидываний моентки - первый бросок - выбор пары (5-10 или 10-20), второй, выбор конверта из выбранной пары.

    И для такого случая формула D-nV мне кажется верной - 0.25 размер выигрыша при смене конверта и 0.25 проигрыша при потере ранее выбранного конверта.

    В жизни я согласен, нельзя сделать полностью равновозможное распределение денег по конвертам (даже у компьютеров есть ограничения в разрядности), но здесь задача "на бумаге", т.е. применения законов реального мира, имхо, неуместны.

    PS. Сейчас ещё читаю предложенную статью в вики но с моим уровнем английского это явно затянется :(

    ОтветитьУдалить
  83. CaXaP, спасибо за тёплые слова.

    Вы пишете «Я считаю, что задачу без потери смысла можно переформулировать так:
    Есть 2 конверта - человек выбирает конверт. После этого, во второй ложат сумму либо меньшую, либо бОльшую.
    ». Вообще-то, если Вы меняете условие задачи, то и ответ может измениться. И для данной задачи он меняется.

    Желание поменять условие задачи вполне понятно, но я хочу обратить внимание, что получилась совсем другая задача. И из её условия как раз очевидно следует, что всегда выгодно менять первый конверт на второй (так как мы с вероятность 50% теряем x/2, но в то же время с той же вероятностью 50% приобретаем x).

    Для исходной же задачи это неверно. Потому что постановка первой задачи не гарантирует, что во втором конверте большая и меньшая суммы будут с одинаковыми вероятностями, а постановка второй задачи (Ваша формулировка) это гарантирует.

    Успехов в освоении теории вероятностей!

    ОтветитьУдалить
  84. Получается, что мне нужно

    1. Поверить (убедиться) что в моей формулировке выигрыш действительно очевиден и если менять ковенрты можно будет бесконечно (и в каждом следующем будет сумма либо 0.5х, либо 2х), то меняющий сможет заработать любое количество денег (т.к. средний выигрыш 0.25).

    2. Согласиться с тем, что моя формулировка и изначальная различны. Но тут уже скорее вопрос "понимания условия" - у каждого он свой. Я всё же считаю, что условие в данном виде подразумевает полную независимость от человеческого фактора, потенциальных границ возможных сумм в конвертах и т.п.
    А Вы, насколько я понял, ссылаетесь на то, что "сферических коней в ваакуме" не бывает, и идеально равновероятно заполнить 2 конверта деньгами не получится...

    Буду думать дальше :)

    В любом случае спасибо, это был очень сильный удар по моему мировоззрению :)

    ОтветитьУдалить
  85. CaXaP, желаю Вам успехов в понимании этой задачи! Рад, что Вы начали осознавать, что Ваше изменение формулировки существенно меняет суть.

    Уточню только, что Ваша фраза «Я всё же считаю, что условие в данном виде подразумевает полную независимость от человеческого фактора, потенциальных границ возможных сумм в конвертах и т.п. А Вы, насколько я понял, ссылаетесь на то, что "сферических коней в ваакуме" не бывает, и идеально равновероятно заполнить 2 конверта деньгами не получится...
    » не совсем корректна.
    Выше было доказано, что неограниченного равномерного распределения не может быть. Дело не в сферичности коней, а в принципиальном несуществовании таких объектов. Поэтому наивно рассчитывать, что в конвертах могут быть неограниченные суммы, (и при этом распределение этих сумм будет равномерным).

    А это значит, что надо исходить из того, что какие-то ограничения есть (пусть мы и не знаем, какие именно). А раз так, то для второго конверта гарантировать свойство равновероятности большей и меньшей сумм нельзя.

    Успехов!

    ОтветитьУдалить
  86. и если менять ковенрты можно будет бесконечно (и в каждом следующем будет сумма либо 0.5х, либо 2х), то меняющий сможет заработать любое количество денег (т.к. средний выигрыш 0.25).

    Отличный пример. Но тогда чтоб вероятность была 0.5 независимо от суммы в открытом, нужно формировать второй после вскрытия первого. И, похоже, никак иначе.

    То есть после вскрытия нами конверта ведущий втайне подкидывает монетку, кладет сумму во второй конверт и предлагает поменять или нет.

    Тут стратегия "меняй пока у него не закончатся деньги" вполне уместна.

    ОтветитьУдалить
  87. D-nV : [Да как же так? Я же выше чётко написал формулу ожидаемой прибыли при таком раскладе (50х50), и показал...]

    Я вроде показывал, что Ваша формула ошибочна. Мог-бы просто сказать, что она неправильная... но решил указать где, и даже провел те-же рассуждения на другом примере и получил неверный ответ. То есть доказал ошибочность Вашего подхода.

    Вы меня игнорируете, да?

    P.S. все это напоминает анекдот "в интернете кто-то неправ".

    ОтветитьУдалить
  88. Антон: Тут стратегия "меняй пока у него не закончатся деньги" вполне уместна.

    Оказывается нет. Если аккуратно посчитать, оказывается, что с каждой сменой матожидание выигрыша от смены конверта скатывается к нулю.

    Значит надо менять конверт один раз и переходить на следующий раунд.

    ОтветитьУдалить
  89. Илья, наконец-то я дождался от вас этих слов! ). "постановка первой задачи не гарантирует, что во втором конверте большая и меньшая суммы будут с одинаковыми вероятностями, а постановка второй задачи (Ваша формулировка) это гарантирует."
    Но в условии первой задачи чётко сказано, что в одном из конвертов сумма денег в два раза больше, чем в другом. Вы полностью и изначально согласны с тем, что шансы вытянуть бОльшую или меньшую сумму в первом конверте равны. Но во втором-то конверте НЕ МОЖЕТ БЫТЬ ничего, кроме меньшей суммы в случае получения в первом конверте бОльшей или бОльшей, в случае получения в первом конверте меньшей. Дальше формула услвной вероятности. В итоге получаются те же равные вероятности.

    И в нашей первой задаче, и в задаче, предложенной СаХаРом (спасибо, что поддержали) вероятности при замене равны. Но это не мешает доказать, что в первом случае замена бессмысленна, а во втором будет выигрыш. В первой задаче нужно рассматривать две пары конвертов, а во второй одну - в этом вся разница.

    В конце нашего спора лишь скажу, что, возможно, не всегда стоит искать ошибку только в чужой позиции. Может быть тут я всё же прав? :)

    Антон, я вас не игнорирую. Просто задача настолько проста, что не нужно лезть в такие дебри. Ну хорошо, пусть там не равномерное распределение. Просто речь идёт про средние значения, точнее, математическое ожидание событий. Что оно будет 50% вы тоже не согласны?

    ОтветитьУдалить
  90. Ну сколько же можно…
    Вам говорят 2+2≠5. Вы в ответ выдаёте «пусть 2+2=5, тогда…» и дальше куча рассуждений с формулами. Расчёт мат. ожидания я даже не читал, он ничего не меняет.

    По формулировкам задач
    Они отличаются принципиально. В первом случае мы имеем дело с одним случайным событием — выбором пары из двух чисел. Пары принципиально не могут быть равноправны. В условии требуется определить по выпавшему числу n определить, к какой паре он относится, (n/2, n) или (n, n×2). Не может быть такого закона распределения, чтобы эти пары были равновероятны. Вернее может быть один забавный случай, если P(A) = 1 при A = (0, 0) и P(A) = 0 в противном случае), но такой вариант нас не интересует. Искомую вероятность нам больше определять не из чего. Формулу условное вероятности здесь использовать нельзя, потому что в ней фигурируют как минимум два случайных события, а у нас только одно.

    Во второй задаче мы имеем дело именно с двумя событиями, причём закон распределения первого нас совершенно не волнует, можно начинать сразу с выбора суммы во втором конверте при фиксированной сумме в первом. Предположение равновероятности этого второго выбора не приводит к ереси, и всё нормально решается.

    ОтветитьУдалить
  91. D-nV писал

    Просто задача настолько проста, что не нужно лезть в такие дебри.
    Где дебри? Если сумма ряда дебри, то тер.вер. дебри в двойне. Потому что он строиться на этом.

    Ну хорошо, пусть там не равномерное распределение. Просто речь идёт про средние значения, точнее, математическое ожидание событий. Что оно будет 50% вы тоже не согласны?
    Не согласен. Именно об этом 70% моих постов.

    ОтветитьУдалить
  92. D-nV, давайте попробуем разобраться с Вашей последней репликой.

    Вы пишете: «Илья, наконец-то я дождался от вас этих слов! ). "постановка первой задачи не гарантирует, что во втором конверте большая и меньшая суммы будут с одинаковыми вероятностями, а постановка второй задачи (Ваша формулировка) это гарантирует."
    Но в условии первой задачи чётко сказано, что в одном из конвертов сумма денег в два раза больше, чем в другом. Вы полностью и изначально согласны с тем, что шансы вытянуть бОльшую или меньшую сумму в первом конверте равны. Но во втором-то конверте НЕ МОЖЕТ БЫТЬ ничего, кроме меньшей суммы в случае получения в первом конверте бОльшей или бОльшей, в случае получения в первом конверте меньшей.
    »
    Да, конечно, это верно. Ошибка в следующем переходе.

    Вы пишете: «Дальше формула услвной вероятности. В итоге получаются те же равные вероятности.».
    Выше я уже отвечал на этоздесь). Разберитесь, что именно Вы вычисляете по формуле условной вероятности (и как её правильно применять), тогда многие вопросы отпадут.

    Ещё раз перечислю верные утверждения:
    1) вероятность того, что в первом конверте будет сумма большая из двух (из тех двух, что лежат в конвертах) - 50%.
    2) вероятность того, что во втором конверте обнаружется сумма меньшая из двух - 50%.
    3) вероятность того, что если в первом конверте x, то во втором будет 2x - какая угодно (это зависит от закона распределения денег по конвертам и от значения x).

    С Вашими словами о том, что «не всегда стоит искать ошибку только в чужой позиции.» я согласен - надо искать ошибку в позициях обоих: и в своей, и в позиции собеседника.

    ОтветитьУдалить
  93. Антон: Тут стратегия "меняй пока у него не закончатся деньги" вполне уместна.

    Оказывается нет. Если аккуратно посчитать, оказывается, что с каждой сменой матожидание выигрыша от смены конверта скатывается к нулю.

    Значит надо менять конверт один раз и переходить на следующий раунд.


    Тут уже я не понял :(

    Имхо, после смены конверта, следующий "раунд" начинается автоматически - смотрят сколько у тебя денег и в зависимости от этого формируют следующий конверт. При таких условиях ожидаемый выигрыш по моим прикидкам так и останется на уровне 25%.

    А по формулировкам, как я понял, разница такая: если выбрать одно число, а затем зависимое от него второе, то вероятности будут равны (моя формулировка). А в Вашей формулировке выбираются сразу взаимозависимые пары на бесконечном множестве и сделать равновероятным выпадение этих пар нельзя...

    Но всё равно для понимания этого, имхо, нужно довольно тесно с ТВ соприкасаться - мне, как просто прослушавшему курс, всё это кажется безумно запутанным и неинтуитивным... Видимо на то он и "парадокс" :)

    ОтветитьУдалить
  94. CaXaP: Имхо, после смены конверта, следующий "раунд" начинается автоматически - смотрят сколько у тебя денег и в зависимости от этого формируют следующий конверт. При таких условиях ожидаемый выигрыш по моим прикидкам так и останется на уровне 25%.

    ну да, но 0.25^n -> 0 при n->inf :)

    Тут нет случайной, независимой от предыдущего раунда, генерации суммы в конвертах. Именно поэтому нельзя считать это отдельным раундом.

    Выхлоп от раунда в целом сохраняется положительным.
    А вот бонус от смены конверта уменьшается с каждой сменой внутри раунда.

    ОтветитьУдалить
  95. Антон, или я Вас не понимаю, или я не согласен с Вашими последними выкладками :)

    Как я понял, СаХаР предлагает рассмотреть такую игру: как только игрок выбрал конверт с суммой х, то во второй с вероятностями 0.5 кладём 2х или х/2. Если игрок после этого просит второй конверт (с суммой y), то ведущий сразу же меняет содержимое первого, кладя в него 2y и y/2 с вероятностью 0.5. И так далее. Если в такой игре постоянно просить поменять конверт, то суммы будут расти.

    Если я правильно Вас понял, то Вы зря считаете предел 0.25^n. На каждой итерации сумма в текущем конверте увеличивается в среднем в 1.25 раз, поэтому надо считать 1.25^n (при n->inf). Тогда всё встаёт на место: чем больше раз поменяем конверт, тем большую сумму выиграем в среднем в игре СаХаРа.

    ОтветитьУдалить
  96. Всем по порядку.
    Xavier.
    "Формулу условное вероятности здесь использовать нельзя, потому что в ней фигурируют как минимум два случайных события, а у нас только одно."
    Вы неправильно понимаете понятие "событие". Событие в данном случае - вытягивание бОльшей либо меньшей суммы. В нашей задаче это несовместимые события. Нужно использовать именно условную вероятность. Как с монеткой (это тут хорошая аналогия) - тоже два несовместимых события. В первой задаче мы подкидываем монетку и всегда её переворачиваем. Во второй мы по очереди кидаем две монетки. Чувствуете разницу? А вероятности-то орла/решки всегда равные!

    Антон, если вы не согласны с базовым утверждением, что каким бы там ни было распределение случайных величин в нашей задаче, матожидание каждой будет равным, то мне просто нечего ответить, извините.

    Илья, выпишу ваши утверждения:
    1) вероятность того, что в первом конверте будет сумма большая из двух (из тех двух, что лежат в конвертах) - 50%.
    2) вероятность того, что во втором конверте обнаружется сумма меньшая из двух - 50%.
    3) вероятность того, что если в первом конверте x, то во втором будет 2x - какая угодно (это зависит от закона распределения денег по конвертам и от значения x).

    На самом деле мы рассматриваем события, поэтому будет звучать немного не так:
    1) Вероятность того, что из первого конверта мы ВЫТАЩИМ меньшую из двух сумм равна 50%.
    2) Вероятность того, что (после того, как мы вытащили из первого конверта меньшую сумму) из второго конверта мы вытащим бОльшую сумму равна сколько? Правильно, 100%. Нельзя же вытащить сумму меньше меньшей, эти события несовместимы ;)

    Это первый вариант развития событий. 50% первого шага. Есть ещё вариант, что из первого конверта мы вытащим сумму, бОльшую из двух, тоже 50%. И, соответственно, вероятность вытащить из второго конверта меньшую сумму будет в этом случае 100%.
    Это второй вариант развития событий.

    3) Бог с ним, без условной вероятности.
    Первый вариант развития событий однозначно приводит к вытягиванию бОльшей суммы. Общая вероятность такого исхода 0,5*1=0,5.
    Второй вариант однозначно приводит к вытягиванию меньшей суммы, общая вероятность развития событий по второму варианту тоже 0,5 (т.е. 50%).
    Что и требовалось доказать: вероятность вытянуть бОльшую сумму во втором конверте (после смены первого) равна вероятности вытянуть меньшую и равна 50%.
    Нужно просто рассматривать СОБЫТИЯ, а не неизвестно что, тогда и вероятности будут известны :)

    Опять не убедил?
    Переложим ваши "верные утверждения" на монетку:
    1) вероятность того, что с одной стороны монетки решка - 50%.
    2) вероятность того, что на другой её стороне обнаружится орёл - 50%.
    3) вероятность того, что если с одной стороны монеты орёл(решка), то на другой её стороне будет решка(орёл) - какая угодно (это зависит от закона распределения орлов и решек по сторонам монетки и от стороны монетки).

    Не правда ли, с моей позиции ваши слова звучат странно? ;)
    С последним утверждением (которое в скобках) я конечно утрировал. Понятно, что к орлу можно привязать сумму Х, к решке - 2Х. Вероятности от этого не изменятся. Хотя, Антон при желании вам наверно может доказать, что и при подкидывании монетки вероятности (и матожидание, если орёл 0, решка 1) не равны 0,5... ;)

    ОтветитьУдалить
  97. D-nV:согласны с базовым утверждением, что каким бы там ни было распределение случайных величин в нашей задаче, матожидание каждой будет равным, то мне просто нечего ответить, извините.

    Вытянуть большую или меньшую суммы - вероятности равны, и эти события несовместные. До того как конверт выбран.

    А вот уже после того, как конверт открыт, вероятности "Вытянуть большую или меньшую суммы" больше не существует, потому что событие свершилось. Теперь эти "вероятности" равны 0 и 1, вот только какое 0, а какое 1 мы не знаем.

    ОтветитьУдалить
  98. Антон
    "Теперь эти "вероятности" равны 0 и 1, вот только какое 0, а какое 1 мы не знаем."
    Начнём с того, что 0,0 и 1,1 тоже несовместные события. Возможны только 0,1 и 1,0. Да, после вытягивания первого конверта всё уже определено. Не важно, видим ли мы, что вытянули в первом конверте, но если мы всегда его меняем, то имеем те же равные вероятности после замены (переворота монетки, если по аналогии).

    ОтветитьУдалить
  99. D-nV, Ваше упорство, безусловно, достойно более удачного применения.

    Вы в очередной раз представляете дело так, как будто спор идёт о том, что раз в первом конверте бОльшая из двух сумма будет с вероятностью 50%, то и во втором конверте бОльшая из двух сумма найдётся с той же вероятностью 50%.

    Вы это можете обосновывать сколько угодно раз, я с этим всё равно буду согласен. Выше я неоднократно с этим соглашался, могу согласиться вновь, если это поможет.

    Поймите, пожалуйста, что это странно выглядит. Вы пишете «Что и требовалось доказать: вероятность вытянуть бОльшую сумму во втором конверте (после смены первого) равна вероятности вытянуть меньшую и равна 50%. Нужно просто рассматривать СОБЫТИЯ, а не неизвестно что, тогда и вероятности будут известны :) Опять не убедил?», как будто кто-то с этим не согласен. Конечно, убедили! Это очевидные пункты, по которым с Вами уже много раз согласились. И аналогия с монеткой более чем понятна, но в данном случае некорректна (речь о Вашем третьем пункте «3) вероятность того, что если с одной стороны монеты орёл(решка), то на другой её стороне будет решка(орёл) - какая угодно (это зависит от закона распределения орлов и решек по сторонам монетки и от стороны монетки)» - это абсолютно не соответствует моему утверждению (потому что, зная текущую сторону монетки, мы знаем и ей противоположную, а в задаче о конвертах мы не знаем, будет там вдвое большая сумма или вдвое меньшая).

    Выше я объяснял уже три раза, что вопрос состоял в другом. Но Вы не разу не написали, что не поняли моих слов (чтобы я конкретизировал, если это требуется), ни разу не написали, что поняли, но несогласны с ними, указав пункты, по которым не согласны (чтобы я мог объяснить эти пункты). Означает ли это, что Вы не хотите понять, а хотите убедить себя, что все вокруг ошибаются? Если нет, то предлагаю разобраться в высказанных ранее возражениях, а не плодить новые однотипные вопросы. Начните с последнего. Поймите, что утверждения (2) и (3) отличаются, разберитесь, с какими из них Вы согласны, а какие не принимаете (и поясните, почему не принимаете). И дальше можно будет продолжить конструктивный разговор, потому что я смогу ответить на чёткие вопросы по этим непониманиям.



    И ещё один важный момент - об уважении: Вы очень неуважительно пишете «Антон при желании вам наверно может доказать, что и при подкидывании монетки вероятности (и матожидание, если орёл 0, решка 1) не равны 0,5... ;)», как будто мы тут соревнуемся, кто более бредовое утверждение выдвинет. Я прошу больше не допускать неуважения к оппонентам, постарайтесь быть корректным!

    ОтветитьУдалить
  100. D-nV
    Вот что бывает, когда формулируешь мысли в сонном состоянии. Хотел показать, что условная вероятность совершенно ни к чему в первоначальной постановке, а в результате написал какую-то чушь. Снимаю из рассмотрения последнее предложение второго абзаца. Можно было бы сформулировать нормально, но совершенно лень.
    Аргументы по делу есть?
    Всё, что вы сейчас пишете, либо основывается на ложных посылках, либо не имеет отношения к исходной постановке задачи, либо и без того очевидно.

    ОтветитьУдалить
  101. Попробую расставить события по местам.

    События:
    Б - выбрать больший конверт
    Б' - выбрать меньший конверт
    K(x) - сумма денег в конвертах равна х
    A(x) - в выбранном конверте х денег

    В - выигрыш при смене конвертов
    В' - проигрыш при смене конвертов

    B(x) - выиграть, при том что в открытом конверте увидели х

    Б и Б' - несовместны и равновероятны.
    {К(3), К(4), ...} - несовместны, вероятность не знаем, но точно не равновероятны.
    A(x) - несовместны

    Б и К(x) - независимы.
    A(x) и К(x) - заведомо зависимы.
    К(3x)=> (A(2x) или A(x))

    P{Б}=P{Б'}=0.5 (не спорю)

    A(x) = ( Б и К(1.5х) ) или (Б' и К(3х))

    B = Б' (независимо от суммы в конвертах и способа раскладывания)

    В(х) = К(1.5х)|A(x)

    B и B(x) - разные события!

    Мат ожидания до вскрытия:
    1. Сумма в первом конверте
    =/sum_х P{A(x)}*x
    =/sum_x (P{К(1.5х)}*P{Б}+P{К(3х)}*P{Б'})*x =/sum_x (P{К(1.5х)}+P{К(3х)})*x/2
    2. Сумма во втором конверте
    =(P{К(1.5х)}*P{Б'}+P{К(3х)}*P{Б})*x =/sum_x (P{К(1.5х)}+P{К(3х)})*x/2
    такая-же как и в первом.
    3. Выигрыш от смены конверта
    =0

    Если конверт выбран, но не вскрыт ожидания будут такими-же как и выше. Потому что Б' = B, а значит P{Б} = P{Б'} = P{B} = (1-P{B}) = 0.5

    За пункт 3 D-nV отчаянно бьется. Так? Ну я и не спорю.

    Мат.ожидания после вскрытия (A(a) случилось):
    1. Сумма в первом конверте
    =a
    2. Сумма во втором конверте
    =P{К(1.5х)|A(a)}а/2+P{К(3х)|A(a)}2а =(a/2)*(1-P{B(a)}) + 2a*P{B(a)}
    =a(1/2 + P{B(a)}3/2)
    3. Выигрыш от смены конвертов
    =(a/2)*(1-P{B(a)}) + 2a*P{B(a)} - a
    =a(P{B(a)}3/2-1/2)

    Теперь про игру с кучей перемен конвертов и заполением второго после первого. Илья Весенний, Вы правильно поняли ее условия.
    Пусть в руках на и-ой итерации C_i-случайная величина. Тогда ожидание в другом конверте M(C_{i+1}) = /sum_x x*P{C_{i+1}=x} = /sum_x x(P{C_i=x/2} + P{C_i=2x})/2 = /sum_x P{C_i=x/2}x/2 + /sum_x P{C_i=2x}2x/4 = M(C_i) + M(C_i)1/4 = M(C_i)5/4 = C_0*((5/4)^i)
    В смысле был не прав.

    ОтветитьУдалить
  102. Уважаемые,

    Ну вы тут нагородили. Горе от ума читали ? :)

    В исходном решении которое преподносится как пародокс, есть ошибка.
    Почему-то в решении предлагается такое пространство равновероятных событий:

    1. X в первом , 2X во втором конверте
    2. X в первом , X/2 во втором конверте

    Но ведь не зависимо от того - вскрыли конверт или нет, сумма денег в обоих конвертах не должна меняться (деньги туда клались до того как был сделан выбор)
    А у авторов она получается разная : 3X в первом случае и 1.5X во втором.

    Правильным будет рассмотрение пространства равновероятных событий:
    1. X в первом , 2X во втором конверте
    2. 2X в первом , X во втором конверте

    Матожидания у обоих конвертов (X+2X)/2 = 1.5X
    Т.е. менять бессмысленно, никакого парадокса нет.

    А в варианте задачи, который предложил CaXaP, суммы уже в этих событиях будут разные и соответственно правомерно применять такое решение.

    ОтветитьУдалить
  103. Roustem, спасибо за краткое и ясное изложение! Вы мастерски и точно формулируете, что должно помочь многим разрешить оставшиеся вопросы.

    Ещё одна хорошая новость - обновилась русскоязычная статья википедии. Сейчас в ней подробно описана сама задача, предложено два объяснения "парадокса", разобраны различные стратегии игры. Рекомендую прочитать всем, кто ещё не разобрался!

    ОтветитьУдалить
  104. Этот комментарий был удален автором.

    ОтветитьУдалить
  105. RayRoko,

    Дайте сначала определение элементарных событий, множества случайных событий и их вероятностей в вашем подходе, иначе все рассуждения бессмыслены, т.к. без этой тройки не существует и самого вероятностного пространства :)

    ОтветитьУдалить
  106. Анонимный06.09.2009, 05:09

    Для меня в этой проблеме нет парадокса. Все проблемы идут из за того, что оперируют понятиями, не вдумываясь в них.
    Симметрия - имеется в виду ествественно симметрия кривой нормального распределения Гаусса: http://ru.wikipedia.org/wiki/%D0%9D%D0%BE%D1%80%D0%BC%D0%B0%D0%BB%D1%8C%D0%BD%D0%BE%D0%B5_%D1%80%D0%B0%D1%81%D0%BF%D1%80%D0%B5%D0%B4%D0%B5%D0%BB%D0%B5%D0%BD%D0%B8%D0%B5
    .
    Симметрия сохранена в том, что когда задача повторяется, и количество пар конвертов стремится к бесконечности, то в каждом втором будет А/2 денег, и в каждом втором будет А*2 денег.

    А интуитивное представление о равпоправности первого конверта, где А рублей, и 2гого, где может быть или А/2 или А*2 рублей, ложно. Доказательство:
    При итерациях (проведениях повторов) задачи и выборе всегда первого конверта среднестатистический выигрыш будет всегда А и только А. не больше и не меншье.
    При итерациях задачи и выборе всегда второго конверта среднестатистический выигрыш будет средним арифметическим сумм конвертов. А так как суммы, в соответствии с нормальным распределением Гаусса, будут нормально распределены между А/2 и А*2, то их средн.арифм. величина и будет 5/4*А., т.е. больше, чем А. И тут нет никакой ошибки. С случае с 10 рублями, постоянная выборка 2го конверта с (5ю или 20ю) рублями даст нам (5+20)/2=12.5=5/4*10 рублей.
    Заметьте, изюминка в постановке задачи, и менять на 2й конверт имеет смысл только тогда, когда максимальная сумма выигрыша больше, чем 1.5А, при минимальной сумме всё той же, 0.5А, так как .5*(.5A)+.5*(1.5A)=A (то есть если во втором конверте или пол-А или полтора-А, то меняя конверт с А на 2й, мы ничего не проиграем и не выиграем при итерациях задачи).

    В английской версии статьи про данный парадокс указано про мнимую необходимость бесконечного обмена конвертов, но в том заключении содержится логическая ошибка, так как содержимое первого конверта нам уже известно, и оно не вероятносто, а уже закреплено каузальностью.

    ОтветитьУдалить
  107. Анонимный07.09.2009, 10:21

    Жаль, что не могу прочитить о задаче на английском, но вы Илья Весенний как то слишком поверхностно мыслите и пытаетесь сманипулировать на динозаврах про вероятность 50%...

    При любых начальных условиях вероятность вытянуть конверт с суммой в 2 раза выше равна 50% и тут нужно просто немного подумать глубже.

    Смотрите, допустим есть некий Якубович, который кладет в один конверт 10 рублей, а в другой 20. Делая это ВСЕГДА.

    Какая вероятность того, что вы вытяните первый или второй конверт на данном этапе? Особого ума не нужно, что очевидно, что эта вероятность 50%.

    А теперь, не заглядывая в конверт скажите, какова вероятность того, что вы вытяните конверт с удвоенной или ополовиненной суммой. Эта вероятность опять же таки 50%.

    Поэтому абсолютно не важно, как кладутся деньги в конверт, хоть рандомно, хоть с одной и той же последовательностью, вероятность 50% и пример с динозаврами выглядит совсем нелепо.

    ОтветитьУдалить
  108. Уважаемый аноним, сейчас есть хорошая версия статьи в википедии на русском языке, поэтому знание английского не так нужно для вникания в этот парадокс.

    На Ваше следующее возражение о вероятностях я отвечал неоднократно выше. Вы совершенно правы, что выбирая первый конверт, мы получим бОльшую из двух сумм с вероятностью 50% (и нет разницы, какое распределени).

    Но дальше мы говорим о другой вероятности - не путайте их. Прочитайте соответствующие комментарии выше - всё станет ясно. Если останутся вопросы, то обращайтесь. Успехов!

    ОтветитьУдалить
  109. Изменится ли количество денег в конвертах после вскрытия первого? Будем считать, что конверты не волшебные, и не изменится. Тогда можно выбрать первый, а потом, не вскрывая первого, выбрать второй. Там ожидаемая сумма будет 5/4 от первой. Результат же не изменится от того, что мы не вскрыли первого конверта. Потом снова первый, не вскрывая второго (по той же логике, в первом ожидаемая сумма уже 25/16). И так до бесконечности.
    А вы говорите, не волшебные ))

    ОтветитьУдалить
  110. Анонимный07.09.2009, 22:53

    Этот комментарий был удален администратором блога.

    ОтветитьУдалить
  111. Уважаемый аноним, я удалил Ваш комментарий, потому что он содержит непечатные выражения. Это недопустимо на данном блоге.

    Но на Ваш вопрос попробую ответить. Вы путаете две вероятности, как я и писал выше.

    Процитирую Вас: «Мы не знаем в каком конверте, какая сумма. Какова вероятность для нас, что в первом конверте большая сумма? - 50%
    Какова вероятность, что во втором конверте большая сумма? - 50%
    » - зачем это повторять? Я с этим уже много раз согласился.

    Не согласился я с тем, что если в первом конверте x, то во втором будет 2x с вероятностью 50%. И выше приведено много объяснений этому (мы не знаем распределения денег по конвертам). Если Вы с ними не согласны, то давайте обсуждать Ваши возражения. А Ваши слова о том, что Вы уверены, что тут тоже будет 50% плохо убеждают.

    Ещё раз: Вы путаете две вероятности, поэтому попадаете в ловушку. Если не можете разобраться в комментариях выше и статье википедии, то попробуйте сформулировать, что именно Вам не ясно - я постараюсь объяснить.

    ОтветитьУдалить
  112. Кстати, Илья, почитал википедию и там, слава богу, упоминаний о вашей "неопределённой вероятности" не увидел. Не знаю, вы ли писали ту статью, но с тем, что там сейчас согласен. Оно нигде не противоречит тому, что говорил тут я.

    ОтветитьУдалить
  113. D-nV, рад, что Вы разобрались во всём по статье википедии. Её писал не я (я только внёс несколько грамматических правок), потому что не хотел порождать лишние сомнения читателей (мол, написал свои завиральные идеи на блоге, а потом принёс их в википедию).

    Но я хочу сделать акцент на том, что "неопределённая вероятность" вовсе не моя - это нормальный научный взгляд на данную задачу.

    Заметьте в статье википедии в разделе "Стохастический случай с известным распределением" как раз есть фраза «Другими словами, чем меньше сумма в открытом конверте, тем с большей вероятностью следует сменить конверт, и наоборот, чем больше сумма в открытом конверте, тем скорее следует оставить этот конверт себе.» (и перед этим ещё разобрано четыре понятных примера).

    Что значит эта фраза и примеры? Что вероятность зависит от того, что именно в первом конверте (и от распределения, конечно). И что "просто так", ничего не зная о распределении, эту вероятность вычислить нельзя (и тем более нельзя считать, что она равна 50%). Об этом я и говорил.

    ОтветитьУдалить
  114. рад, что Вы разобрались во всём по статье википедии
    Ничего подобного я не говорил. Разобрался я сам, ещё с первого своего поста. Я просто сказал, что я согласен с тем, что написано в википедии. А там не написано этого бреда, что вероятность вытащить из второго конверта (всегда меняя первый) большую (меньшую) сумму не равна 50%.

    Что касается случая, когда мы выбираем, менять ли нам конверт в зависимости от выпавшей суммы - это совсем другая задача и там, естественно, сложно посчитать вероятность. Нужно хотя бы точно определить условия смены. Но мы же тут говорили совсем о другом, о БЕЗУСЛОВНОЙ смене конверта.

    Короче, раз уж мы до сих пор ни до чего не дошли, останемся каждый при своём мнении. Ничего страшного. Важно, что хоть выводы у нас совпадают :)

    ОтветитьУдалить
  115. D-nV, Вы пишете «А там не написано этого бреда, что вероятность вытащить из второго конверта (всегда меняя первый) большую (меньшую) сумму не равна 50%.», как будто это написано у меня в блоге. Я тоже считаю данное высказывание некорректным, поэтому Ваши слова считаю оскорбительными.

    Вы в очередной раз позволяете себе приписывать человеку слова, которые он не говорил и которые считает ошибочными, а потом называете "его" высказывание бредовым - это просто недостойно в приличном обществе.

    Я не хочу Вас разубеждать по сути, но категорически против такой формы ведения спора у себя в блоге. Пожалуйста, воздерживайтесь от подобных манипуляционных техник в следующий раз.

    ОтветитьУдалить
  116. Анонимный08.09.2009, 21:03

    Илья Весенний, ясно же, что D-nV - тролль! Не надо ему отвечать! Ему нравится задавать одинаковые вопросы, игнорируя Ваши ответа, поэтому хватит поддерживать эту игру.

    ОтветитьУдалить
  117. Илья, будем считать, что я вас не так понял. Если эти слова вам не принадлежат, извините.

    И тут приходит Анонимный пользователь со своим мнением по вопросам "кто есть кто" и "что делать" :) Где я задавал вопросы? Я просто высказывал свою точку зрения, наверно слишком настойчиво.

    ОтветитьУдалить
  118. Уважаемый аноним, нельзя обвинять человека просто так. Мне кажется, что D-nV искренне не понимает разницы между двумя утверждениями, которые я приводил (например, второе и третье в комментарии выше). А если их не различать (несмотря на мои настойчивые предложения понять, что они о разном), то можно очень долго путаться и иметь иллюзию, что люди вокруг говорят бред. Я думаю, если D-nV искренне хочет разобраться, то он постепенно сформулирует, что именно ему непонятно. Поэтому, уважаемый аноним, я прошу Вас больше не действовать в такой оскорбительной манере.

    D-nV, извинения приняты. Жаль, что в Вас прилетел этот комментарий от анонима, прошу Вас не обижаться на такие слова. И надеюсь, что они больше не повторятся.

    Однако я понимаю, почему у анонима сложилось такое впечатление - Вы игнорируете вопросы, которые задают Вам (в частности, я несколько раз уже просил Вас пояснить, как Вы сделали некоторые выводы, но Вы или не читаете чужие комментарии, или не считаете нужным отвечать на вопросы) - а это мешает разобраться в сути.

    В любом случае, спасибо за интересную дискуссию :)

    ОтветитьУдалить
  119. Сложилось ощущение, что пора дать правильный ответ :)
    Сгенерируем случайное число А. Сгенерируем Б, так, что с вероятностью 50% оно в 2 раза больше А, а с вероятностью 50% в два раза меньше. Мат. Ожидание для Б – 1.25 мат. ожидания для А. Если сгенерировать последовательности и посмотреть, то во второй (Б), значения в среднем будут больше в 1.25 раз. А теперь перемешаем А и Б с вероятностью 50 на 50. Вот теперь мы получили нашу задачу с конвертами. Выясняется, что выбрав конверт и увидев сумму Х, возможны два исхода:
    1. 50% мы выбрали А и ожидание во втором конверте это 1.25, увиденной суммы.
    2. 50% мы выбрали Б и ожидание во втором конверте 0.8, увиденной суммы (1/1.25), но сама увиденная сумма в среднем в 1.25 больше чем в случае 1.
    Мат. ожидание дохода от смены конвертов = 0.5*(1.25-1)+1.25*0.5*(0.8-1)=0
    Суть таких задач не доказать, что в рассуждениях есть ошибка, приведя альтернативные рассуждения, а найти ошибку в доказательстве, составляющем суть парадокса. В формулировке нашего парадокса ошибка в расчете ожидания. Хотя по условию предполагается, что конверты не «сгенерированы», а перемешаны, в расчете пропущено второе слагаемое. А ожидание дохода выродилось до 1*(1.25-1).
    С уважением Алексей Редозубов.

    ОтветитьУдалить
  120. Alexey, что я и твержу с первого поста :)

    ОтветитьУдалить
  121. Alexey, спасибо за ещё одно верное объяснение, показывающее нулевую выгоду от тактики "всегда меняй конверт". Вы очень коротко и ясно описали свои рассуждение, что позволит многим легко их понять.

    Отмечу, что выше этот взгляд на задачу уже был предложен неоднократно, только не с таким усложнением условия.

    Например, D-nV разбирает случай двух конвертов с суммами A=C и B=2C, что позволяет ему вести более простые вычисления после. Ваша модель (в которой B=2C или B=C/2 с равными вероятностями) тоже эквивалентна условию, поэтому её тоже можно корректно использовать, но с ней считать приходится чуть больше :)

    Впрочем, чем больше правильных объяснений, тем проще найти наиболее понятное для конкретного человека - все люди разные, поэтому иногда более сложное нам может показаться более понятным (из-за индивидуальных особенностей). Ещё раз благодарю за ясное и чёткое объяснение.

    D-nV, заметьте, с Вашими верными выкладками никто не спорил. Но были другие верные рассуждения (не противоречащие Вашим), которые Вы не очень понимали, поэтому задавали вопросы (то есть формулировали ошибочные утверждения о том, что кто-то заблуждается), а я пытался их для Вас прояснить - не более того.

    Тема различных адекватных взглядов на один вопрос очень сложна, но и очень интересна для меня. Математика порой позволяет одну задачу решить совершенно разными способами, что открывает большие возможности для понимания отличающихся подходов, вникания в рассуждения других людей, осознания особенностей работы собственного мозга. И здорово, что эти эффекты можно видеть даже на простых задачах - минимального образования достаточно, чтобы почувствовать эту красоту :)

    ОтветитьУдалить
  122. Собственно хотелось акцентировать внимание на том, что не сложно доказать бессмысленность смены конвертов. Достаточно воспользоваться доводом о симметрии ситуаций. Но суть задачи найти ошибку в исходной логике. Этого и не было явным образом сделано в предыдущих постах. Для примера сути парадоксов - "найти ошибку в рассуждениях", предлагаю знаменитый парадокс узника.


    Узник, приговоренный к высшей мере наказания, однажды в воскресенье был вызван к начальнику тюрьмы (честнейшему человеку, никогда не обманывающему даже самых злейших врагов общества). Начальник сказал: "Вас казнят на следующей неделе, но в какой именно день, я вам не скажу. Вы узнаете об этом только утром в день казни".

    Через некоторое время узник обрел способность нормально размышлять. Он немедленно сделал вывод, что казнить в следующее воскресенье его не могут: ведь тогда еще в субботу вечером будет ясно, что для казни остался один-единственный возможный день. Это будет означать, что узнику стал известен день казни еще до наступления этого дня, а такого не может быть по определению - ведь начальник никогда не лжет!
    "Иными словами, последний возможный день для моей казни, - думал узник, - это суббота. Но тогда если меня не казнят в пятницу утром, то к вечеру пятницы я тоже буду точно знать день, когда меня должны казнить. Однако начальник сказал, что о дне казни я узнаю лишь утром, а не накануне вечером, - значит, и в субботу меня не могут казнить тоже".

    Последовательно рассуждая, несчастный исключил из рассмотрения пятницу, четверг, среду, вторник и понедельник. В итоге он пришел к выводу, что его вообще не могут казнить, поскольку ни для одного дня недели условия, сформулированные начальником тюрьмы, не выполняются.
    Однако в среду за ним пришли и это было для него полной неожиданностью. В чем ошибка в рассуждениях узника?

    ОтветитьУдалить
  123. Исчерпан ли парадокс двух конвертов?

    Примем по итогам обсуждения:
    Интервал, в котором находятся возможные величины, ограничен.
    Возможные величины неравновероятны.
    Вопрос: насколько неравновероятны возможные величины?
    И насколько существенно это влияет на выбор стратегии?

    Предположим, что организаторы парадокса ответственно подошли к составлению раскладов и делают это по алгоритму.
    Алгоритм.
    1- Составляем нарастающую геометрическую прогрессию,
    2- где первое число равно 1, второе 2, третье 4 и т.д..
    3- а последнее число равно 2 в тысячной степени
    4- Вообще, последнее число может быть любым. Назовем это последнее число "Гугол".
    5- Исходя из этой прогрессии, назначим расклады в парах конвертов:
    6- {1;2},{2;4},{4;8},{8;16}...{Гугол/2; Гугол}.
    В итоге, в данном случае, получаем 1000 раскладов с 1000 различными числами, для 998 из этих чисел существуют два и только два расклада. В одном из этих раскладов выбранное число больше другого в два раза, а в другом меньше в два раза.
    Кроме двух крайних чисел прогрессии, каждое из которых присутствует только в одном раскладе.
    7- Составляем убывающую геом. прогрессию,
    8- где первое число равно Гугол-1, второе (Гугол-1)/2 и т.д.,
    9- а последнее – первое же вычисленное значение менее 1.
    10- Исходя из этой прогрессии, назначим расклады в парах конвертов совершенно аналогично п.6

    Сколько раз мы повторим пункты 7...10, меняя первое значение прогрессии на Гугол-2, Гугол-3 и т.д., несущественно. В итоге мы получим множество пар конвертов, с ограничением максимума. В 99,8% из этих пар, после открытия одного конверта в закрытом останутся равновероятными большая или меньшая суммы. Во всех этих 99,8% случаях выгодна стратегия "открыть первый, а взять второй". Еще в 0,1% случаев, когда в первом конверте менее 1, эта стратегия безусловно верна.

    Тут можно заметить, что размещение величин в заданном интервале не является равномерным. Однако, в задаче не указывается, что означает или, что требуется равномерное размещение. Если заданный интервал отобразить на логарифмической шкале, то размещение величин будет вполне равномерным.

    ОтветитьУдалить
  124. Alexey, спасибо за приведение парадокса узника. Ошибка в рассуждениях авторов исходной статьи была явно показана в самой заметке - было объяснено, что в произвольном случае мы не можем считать вероятности сумм 2x и x/2 равными, поэтому их рассуждения некорректны. Но спасибо за Ваш вклад в чёткое формулирование проблемы - я думаю, это многим поможет.

    eLectric, спасибо за интересный пример! Я думаю Вы понимаете, что тактика "всегда меняй первый конверт на второй" сломается, когда участник случайно вытянет в первом конверте Гугол - максимальную возможную сумму. Пользуясь своим правилом, он заведомо обменяет её на вдвое меньшую, потеряв Гугол/2 - тем самым, равновесие системы восстановится :)

    Естественно, если игрок знает максимально возможную сумму, то он не станет делать таких глупостей. Однако это уже другая задача. В любом случае, спасибо за красивый пример!

    ОтветитьУдалить
  125. Илья
    "Ошибка в рассуждениях авторов исходной статьи была явно показана в самой заметке - было объяснено, что в произвольном случае мы не можем считать вероятности сумм 2x и x/2 равными, поэтому их рассуждения некорректны."
    Если что, я с первого поста продолжаю утверждать, что ошибка там есть, но не в этом. Вероятность сумм 2х и х/2 равна. Ошибка в статье в том, что они забывают про первый конверт. Они говорят, что "если вытащили 10 рублей, то во втором 5 или 20 рублей". На самом деле, нужно добавить к рассуждению "и в первом конверте мы могли вытащить 5 или 20". В самом первом посте я показывал, как компенсируется ожидание в таком случае.
    А вы всё про 50%... Не знаю уже, какими словами вам объяснять.
    Ладно, попробую очередной раз.

    Введём два события: "выигрыш" - когда мы вытягиваем больший из двух конвертов и "проигрыш", когда мы вытягиваем меньший конверт.
    Вы согласны с тем, что при вытягивании первого конверта вероятности этих двух событий равны. Но вы не согласны, что при безусловной замене конверта вероятности остаются равными. НО если произошел выигрыш с первым конвертом, то при его замене безусловно произойдёт проигрыш. Если произошел проигрыш с первым конвертом, то при замене безусловно произойдёт выигрыш. Т.к. изначально вероятность была 50%, то при безусловной замене она сохранится. Как это можно не понимать? А если вероятность выигрыша при замене конверта равна вероятности проигрыша, то во втором конверте с РАВНОЙ вероятностью может быть бОльшая или меньшая сумма.
    Так всё просто. Спор ни о чём.

    ОтветитьУдалить
  126. > Вы согласны с тем, что при вытягивании первого конверта вероятности этих двух событий равны.
    Разумеется
    >Но вы не согласны, что при безусловной замене конверта вероятности остаются равными.
    При безусловной замене согласен.
    >НО если произошел выигрыш с первым конвертом, то при его замене безусловно произойдёт проигрыш. Если произошел проигрыш с первым конвертом, то при замене безусловно произойдёт выигрыш.
    Да.
    >Т.к. изначально вероятность была 50%, то при безусловной замене она сохранится.
    Да. 

    Но как все эти факты соотносятся с исходными словами Ильи? Где он говорил про безусловную замену?
    Вероятность зависит от наших знаний о событии. Открыв конверт и увидев сумму, мы получили новые знания. Если при бюджете в миллион в первом конверте оказалось 700 тысяч, то шансы выиграть от смены конверта резко падают до нуля. Безусловная замена просто отбрасывает эти знания, что недопустимо. В задаче безусловная замена не вводится изначально, а получается как абсурдный вывод, и цель как раз и состоит в том, чтобы определить, какие заключения привели к такому выводу. Строить какие-то заключения вокруг абсурда бесполезно.

    ОтветитьУдалить
  127. [b]Илья Весенний, [/b],
    Спасибо за внимание.
    > Естественно, если игрок знает максимально возможную сумму, то он не станет делать таких глупостей.
    ОК. По условиям, он не знает этой Smax. Но в подавляющем числе случаев выигрывает 1,25, если применяет стратегию "возьми второй". Если же в первом конверте окажется больше, чем Гугол/2, что очень маловероятно, то он потеряет сразу от четверти до половины Гугола. (что, как вы сказали, сохраняет равновесие системы). Тогда стратегия должна заключаться в том, чтобы уверенно выбирать второй конверт, когда в первом конверте относительно маленькие деньги (получая выгоду 1,25) и лучше не рисковать, когда в первом относительно большие деньги (чтобы не потерять "всё нажитое непосильным трудом" оказавшись под Гуголом).
    Что полностью согласуется с выводами Эббота и стратегией Ковера.

    ОтветитьУдалить
  128. D-nV, Вы наверняка тоже видели школьных и университетских преподавателей, которые отвечают не на тот вопрос, который задаёт студент (и на который хочет получить ответ), а на совсем другой вопрос (кристально понятный спрашивающему). Что печально, если студент набирается наглости повторить вопрос, то получает повтор известного ему ответа, а не нужные сведения.

    Наша с Вами беседа напоминает мне о таких педагогах. Я неоднократно спрашивал Вас, почему Вы из того, что вероятность одного события 50% делаете вывод, что вероятность другого события тоже 50%. Приведите доказательство хоть один раз или признайте, что Вы в это просто верите, а доказать не можете.

    Вы пишете «Не знаю уже, какими словами вам объяснять.», как будто я ранее не соглашался (опять же, неоднократно) с верными элементами Ваших выкладок. Не надо повторять вновь и вновь то, с чем читатели уже согласились! Ответьте на один вопрос:

    Почему из того факта, что «при вытягивании первого конверта вероятности выигрыша и проигрыша равны» следует, что «наличие в первом конверте суммы x, приводит к тому, что вероятности сумм 2x и x/2 во втором конверте тоже равны»?

    Приведите честное доказательство, тогда я смогу найти в нём ошибку, чтобы Вас разубедить (или соглашусь с его справедливостью - тогда поменяю свою позицию). Или не приводите, тогда мы поймём, что Вы не хотите вникнуть в задачу, а хотите просто убедить нас в чём-от, повторяя одно и то же много раз. Но здесь не политика, поэтому от многократного повторения утверждение не становится истинным.

    И ещё Вы зачем-то опять приписываете мне те слова, с которыми я не согласен категорически: Вы пишете «Вы согласны с тем, что при вытягивании первого конверта вероятности этих двух событий равны. Но вы не согласны, что при безусловной замене конверта вероятности остаются равными.» (как будто я не писал ранее, что считаю их равными). Не надо утверждать того, что я не говорил, потому что это некорректный манипуляционный приём в споре. Надеюсь, Вы используете его случайно, а не с целью превратить конструктивную дискуссию о математике в соревнование по перевиранию слов оппонента. Постарайтесь вести разумный диалог, иначе мы не сможем прийти к общему пониманию.

    ОтветитьУдалить
  129. Предлагаю переименовать "парадокс двух конвертов" в "задачу трех". Так как в соответствии условиями задачи, получив на руки конверт с любой суммой, человек должен немедленно забыть о нем, и начинать выбор между еще двумя конвертами, в которых суммы в два раза больше и меньше первого. Причем первый "наугад" выбранный конверт всегда содержит промежуточную сумму. Удивлен, что никто не обратил внимание на некорректную постановку задачи.

    ОтветитьУдалить
  130. abc, задача, которую предложили Вы, не совпадает с исходной задачей (её выше уже предлагал так переформулировать CaXaP, но потом согласился, что существенно изменил её смысл.

    А условие исходной задачи вполне корректно и из него не следует, что «получив на руки конверт с любой суммой, человек должен немедленно забыть о нем». Если у Вас остались вопросы о задаче, я буду рад ответить на них.

    ОтветитьУдалить
  131. Примечательно, что никто, нигде, никогда не говорит о сущности вопроса.
    Есть, конечно, люди, которые не понимают сам парадокс. Те, кто его понял, благополучно в нем разобрались. И всё. Довольны и забыли. Но дело не в парадоксе. Парадокс только локомотив поп статьи о работе Эббота.
    // Вот другой парадокс - профессор математики, специалист по теории информации не один год занимается проблемой, которая тут всем кажется несложным парадоксом из теорвера//
    Задача касается поиска стратегии в условиях неопределенности.
    Уточнение условий:
    Предел для возможных сумм существует (иначе задача противоречива.
    Возможные суммы - целые числа (ведь речь о деньгах положенных в конверт)
    Наиболее оптимальное и непротиворечивое распределение чисел, имхо, такое: в первой половине интервала от 1 до Предел/2 последовательность натуральных чисел. Во второй половине интервала от Предел/2 до Предел - четные числа.
    Игрок предел не знает.

    Если бы игрок знал предел, то стратегия простая -
    Если попадется число из второй половины интервала - однозначно "не менять"
    Если попадется число из первой половины интервала, нечетное - однозначно "менять"
    Если попадется число из первой половины интервала, четное - тут уже неоднозначно. Но если "менять", то в среднем такая стратегия обеспечивает выйгрыш на 1,25 от первого конверта.

    Но условие, все таки, в том, что игрок предела не знает.

    ОтветитьУдалить
  132. Анонимный17.09.2009, 21:27

    Безусловно, лучший комментарий дал onederey 07/09 в 17:14:

    "...Тогда можно выбрать первый, а потом, не вскрывая первого, выбрать второй. Там ожидаемая сумма будет 5/4 от первой. Результат же не изменится от того, что мы не вскрыли первого конверта. Потом снова первый, не вскрывая второго (по той же логике, в первом ожидаемая сумма уже 25/16). И так до бесконечности".

    Это, по-моему, самый элегантный и убедительный способ показать всю наивную ошибочность стратегии "всегда бери второй".

    ОтветитьУдалить
  133. Анонимный17.09.2009, 21:54

    Что же касается самой задачи о двух конвертах, то её условия следует формулировать более корректно, а именно - дополнить следующими вводными:

    - интервал распределения конечен;
    - пределы этого интервала играющему изначально неизвестны.

    Таким образом, решение задачи сводится к построению динамической итерационной стратегии, которая будет корректироваться с каждым раундом.

    И надо сказать, что в таком виде задачу уже тривиальной не кажется..

    ОтветитьУдалить
  134. >Это, по-моему, самый элегантный и убедительный способ показать всю наивную ошибочность стратегии "всегда бери второй".
    Зачем показывать настолько очевидную вещь?

    >а именно - дополнить следующими вводными
    Да, это одна из возможных формулировок.

    ОтветитьУдалить
  135. eLectric, Вы предложили интересное расширение условия, однако необходимо отметить, что грамотный игрок за некоторое количество туров заметит, что числа менее Предел/2 выпадают в два раза чаще, чем остальные (от Предел/2 до Предел). Так он может с высокой точностью выяснить, чем равен Предел, что позволит выбрать грамотную стратегию на следующие туры. Даже если игрок не знает правила того, кто кладёт деньги в конверты, то он его может "почувствовать", так как люди имеют интуицию :)

    pe3yc, Ваше дополнение к условию понятно. Но и исходная задача вполне корректна. Поэтому правильнее будет говорить не «её условия следует формулировать более корректно», а «можно сформулировать и другую интересную задачу».

    Xavier, очевидные вещи тоже надо формулировать, так как очевидность - субъективная штука. Кто-то всё понимает в одной области знаний, но теряется в другой, а кто-то совершенно наоборот. Если хочется быть понятным для обоих, то не надо стесняться говорить очевидные вещи.

    ОтветитьУдалить
  136. Анонимный18.09.2009, 11:38

    Илья, вы жа сами неоднократно в этом треде говорили о том, что задача в исходной постановке некорректна, поскольку предполагает существование равномерного распределения бесконечной случайной величины. Почему же теперь утверждаете, что она и в такой постановке была корректной?

    ОтветитьУдалить
  137. pe3yc, я говорил, что наивен человек, считающий равномерной любую случайную величину, если она не определена в условии. А если она не равномерна, то проблем нет (и в услоии равномерность не гарантируется).

    Выше Вы можете найти такие утверждения: «при отсутствии априорной информации, по принципу максимума энтропии будет вполне законным будет предположить равномерное распределение». Я не хочу спорить о том, имеет человек право домысливать условие или не имеет, если хочет решить какую-то задачу корректно. Но мне надо было показать, что если начать выдумывать равномерность в данной задаче, то получится рассуждение о несуществующем объекте - а для него можно доказывать что угодно.

    ОтветитьУдалить
  138. Про очевидность я заговорил потому, что никто не оспаривал ошибочность стратегии, вопросы были о том, почему к этой стратегии привели рассуждения.

    ОтветитьУдалить
  139. Распределения вероятностей... хорошо...
    Илья, если вы не согласны с тем, что вероятности заменить сумму на бОльшую или меньшую равны, то вы не можете утверждать и то, что вероятности вытянуть в первом конверте бОльшую или меньшую сумму равны. Т.к. это зависимые события. С этим вы тоже будете спорить?

    ОтветитьУдалить
  140. D-nV, хорошо, что Вы вернулись, но печально, что опять фантазируете. Из-за этого Вы не можете ответить на простой вопрос, который я задавал выше не один раз: почему из верности одного утверждения (с которым я согласен) следует, что верно другое утверждение (с которым я не согласен)? Или докажите его, или признайте, что не можете это сделать!

    Вы верите, что это очевидно, но не можете доказать, правильно? Если да, то это не вопрос науки, а вопрос религии.

    Я доказал выше, что второе утверждение ложно, но Вы не хотите этого замечать, а почему-то приплетаете сюда третье утверждение (с которым я тоже согласен, потому что оно корректно). Это делается, чтобы кого-то запутать? Или Вы так запутываете себя? Перестаньте приписывать мне слова, которые я не говорил, ответьте прямо на мой вопрос. Только тогда мы сможем разобраться с этой задачей. Иначе конструктивного диалога не получится.

    Xavier, я понял Вас. Спасибо за поддержку в дискуссии!

    ОтветитьУдалить
  141. Анонимный19.09.2009, 05:30

    D-nV, всё правильно Илья говорит, прочитайте статью http://www.maa.org/devlin/devlin_0708_04.html
    В ней написано, что причина парадокса в том, что мы неправильно считаем два события равновероятными. А Вы снова и снова пишете, что уних 50% вероятность, как будто читать не умеете o_O

    ОтветитьУдалить
  142. Уважаемый аноним, спасибо большое за дельную ссылку. Действительно, в предложенной статье чётко и подробно объяснено, что если в конвертах суммы A и 2A, а в первом конверте X, то вероятности того, что X=A и X=2A не обязательно равны 50% (это зависит от X и от распределения случайной величины A).

    И из этого прямо следует, что если в первом конверте X, то мы найдём во втором конверте 2X и X/2 тоже не обязательно с равными вероятностями.

    Ещё раз благодарю за ссылку, но прошу в следующий раз воздерживаться от подобных оскорбительных высказываний в адрес оппонентов.

    D-nV, прошу Вас не обижаться на слова, которые могли показаться неприятными. Напишите, пожалуйста, поняли ли Вы рассуждения из статьи или что-то осталось не очень ясным.

    Если вопросы ещё есть, то, пожалуйста, чётко сформулируйте утверждение, которое не принимаете и хотите опровергнуть. И только после этого приводите доводы за или против. Это позволит избежать путаницы. Заранее благодарю.

    ОтветитьУдалить
  143. Анонимный21.09.2009, 14:54

    Добрый день. Очень увлекательная дискуссия!
    Хочу добавить свой вариант решения и задать вопрос.

    Вариант:
    Пусть А - минимальная сумма в конверте (т.е. в одном конверте А, в другом 2*А);
    X - сумма в первом конверте;
    Y - сумма во втором конверте;

    1е уравнение: X + Y = 3*А;

    Теперь, так как мы осуществляем выбор случайным образом (выбираем 1й конверт) по теории вероятностеЙ 2е уравнение: X = (А + 2*А)/2 = 1,5*А;

    Размышляя над тем, брать или не брать ещё один конверт, подставим результат 2 уравнения в 1 уравнение:
    Y = 3*A - X = 3*А - 1,5*А = 1,5А !!!
    Вывод: В среднем сумма X и Y = 1.5А, т.е. одинакова.
    Имеется прямая зависимость между X и Y.

    Вопрос:
    Почему мы не можем второй раз прибегать к теории вероятности? И, соответственно, пользоваться уравнением 3: Y = (X/2 + 2X)/2 ? Не из за того ли, что уже имеется прямая зависимость между X и Y?

    ОтветитьУдалить
  144. Уважаемый аноним, первое уравнение верно и понятно, но что означает Ваше второе уравнение? Я бы понял запись «X=A или 2A».

    Почему мы не можем второй раз прибегать к теории вероятности? И, соответственно, пользоваться уравнением 3: Y = (X/2 + 2X)/2
    Это два разных вопроса. Теорией вероятностей мы пользоваться можем, но написанное Вами третье уравнение ей противоречит.

    В предыдущем комментарии как раз подробно объяснено, почему некорректно считать равными вероятности X/2 и 2X.

    ОтветитьУдалить
  145. Анонимный22.09.2009, 08:57

    "но что означает Ваше второе уравнение? Я бы понял запись «X=A или 2A»."
    Второе уравнение означает, что при бесконечном "выбирании" ПЕРВОГО конверта в среднем мы получим сумму (X + Y)/2 = (А + 2А)/2 = 1,5А
    Разве нет?
    Вопрос: При каких условиях мы можем пользоваться теорией вероятностЕЙ?

    ОтветитьУдалить
  146. Уважаемый аноним, выше я Вам ответил, что теорией вероятностей мы можем пользоваться всегда, когда делаем это корректно.

    ОтветитьУдалить
  147. Анонимный22.09.2009, 09:33

    "некорректно считать равными вероятности X/2 и 2X" - некорректно мне тоже кажется. Только потому, что это вообще НЕ вероятности!Вероятность "сработала" при выборе первого конверта. В выборе "менять/не менять" нет вероятности? Почему? (статью (http://www.maa.org) не читал, не умею :))

    ОтветитьУдалить
  148. Анонимный22.09.2009, 09:42

    Всё таки к вопросу о втором уравнении. То есть при выборе первого конверта сумма X в среднем не будет равна 1,5*А ? То есть, я не с равным успехом выберу большую и меньшую суммы? То есть вероятнее, что выберу большую? Или меньшую? Извините, но с этим не могу согласиться.

    ОтветитьУдалить
  149. В выборе "менять/не менять" нет вероятности?
    Я не понимаю этот вопрос. Что значит "нет вероятности"?

    То есть при выборе первого конверта сумма X в среднем не будет равна 1,5*А ?
    Конечно, будет.

    (статью (http://www.maa.org) не читал, не умею :))
    Тогда рекомендую статью в русской википедии - там тоже хорошо написано.

    ОтветитьУдалить
  150. Анонимный22.09.2009, 10:47

    Нет вероятности значит, что Y - не случайное число! Y = 3*A - X = 3*A - 1.5*A = 1.5*A. То есть Y точно определено после первого предположения о сумме X в первом конверте (Что X = 1,5А).
    Я так понимаю, обман в задаче заключен в применении теории вероятностей к величине Y, которая уже определена. А решение задачи - доказательство, что нельзя применять ТВ в указанном случае.

    Моё решение: Величина Y имеет единственное значение (Y = 1,5А), а не "любое случайное значение". Так что мы не можем определять Y по теории вероятности. Вот так.

    ОтветитьУдалить
  151. Уважаемый аноним, теория вероятностей - строгая наука, поэтому её следует применять корректно. Иначе можно будет доказывать сколь угодно неверные утверждения.

    ОтветитьУдалить
  152. Уважаемый аноним,
    Есть или нет какой-либо вероятности при открытии второго конверта - можно смоделировать. Т.е. попросите кого-либо подготовить для вас два конверта согласно правилам "парадокса двух конвертов". Тогда, после открытия первого конверта, вы увидите, что точно не знаете, что будет находится во втором.
    Хотя, совершенно точно можно сказать, что во втором конверте не будет 1,5*А.

    ОтветитьУдалить
  153. Анонимный23.09.2009, 08:47

    "вы увидите, что точно не знаете, что будет находится во втором": Отчего же? Я ТОЧНО БУДУ ЗНАТЬ, что во втором будет находиться в среднем такая же сумма, что и в первом (1,5*А).

    "Хотя, совершенно точно можно сказать, что во втором конверте не будет 1,5*А":
    Сказать можно всё что угодно. А вот доказать? Пожалуйста, обоснуйте свою точку зрения...

    Или проще. Укажите пожалуйста, с каким из пунктов в моём доказательстве (выше) вы не согласны?

    ОтветитьУдалить
  154. "Я ТОЧНО БУДУ ЗНАТЬ, что во втором будет находиться в среднем такая же сумма, что и в первом (1,5*А)."
    Знать "среднее возможное", это вовсе не "точно буду знать".
    После открытия первого конверта точно мы знаем только, что во втором конверте ЛИБО в два раза меньше, ЛИБО в раза больше, чем в первом конверте. Мы знаем точно только ЛИБО-комбинацию из двух возможностей. А каждый из этих вариантов возможен с некоторой вероятностью.

    ОтветитьУдалить
  155. Анонимный23.09.2009, 12:10

    "Знать "среднее возможное", это вовсе не "точно буду знать"":
    Отчего же? Зная среднее возможное я ТОЧНО БУДУ ЗНАТЬ, что при бесконечном количестве игр я получу что в первом конверте, что во втором именно СРЕДНЕЕ ВОЗМОЖНОЕ (1,5*А). Совершенно ТОЧНО.

    "После открытия первого конверта точно мы знаем только, что во втором конверте ЛИБО в два раза меньше, ЛИБО в раза больше, чем в первом конверте":
    Ага. Точно.
    Y = 2*X, когда X = A либо
    Y = X/2 когда X = 2*A

    Y = 2*A когда X = A либо
    Y = А когда X = 2*A

    Ну что, возьмём среднее возможное значение?

    Y = (2*A + A)/2 = 1.5*A когда
    X = (A + 2*A)/2 = 1.5*A

    Однако, если хотите так рассуждать, можно и так. Результат всё тот же. :)

    ОтветитьУдалить
  156. Уважаемый аноним, Вы пишете «Зная среднее возможное я ТОЧНО БУДУ ЗНАТЬ, что при бесконечном количестве игр я получу что в первом конверте, что во втором именно СРЕДНЕЕ ВОЗМОЖНОЕ (1,5*А)», как будто A является постоянно величной. Но по условию задачи А может меняться.

    Если бы A не менялась от игры к игре, то было бы слишком просто - люди бы всегда меняли свой первый конверт с X=A на второй, но никогда бы не меняли конверт с X=2A. И получали бы максимальную прибыль.

    Но игрок не знает A, поэтому, найдя конкретные 10 рублей в первом конверте он не знает, что будет во втором - 5 или 20.

    ОтветитьУдалить
  157. Анонимный23.09.2009, 14:11

    Что же. Попробую доказать, что А (минимальная сумма в конверте)не может меняться, если Вас это смущает.
    Мы знаем, что по условию задачи А - это любое положительное число.
    Запишем:

    0 < A < +бесконечность;

    Для бесконечного количества игр определим, какое же в среднем А будет во всех этих играх (по теории вероятностЕЙ):

    Асред = (0 + +бесконечность)/2 = +бесконечность/2 = +бесконечность;

    Так что А - это для бесконечного количества игр величина постоянная!

    Асред = +бесконечность;

    Пусть вас не путают отдельные игры с 10$, 20$ ...
    Они призваны как раз чтобы запутать. Но если брать СТАТИСТИКУ БЕСКОНЕЧНОГО КОЛИЧЕСТВА ИГР (как в условии задачи), то получается так, как я описываю.

    ОтветитьУдалить
  158. Уважаемый аноним, Вы очень интересно находите среднюю величину - берёте полусумму из наименьшего и наибольшего возможных значений.

    Если бы так все делали, то средняя зарплата в стране была бы гораздо выше - сложили бы зарплату библиотекарши (около 2-5 тысяч рублей) и зарплату топ-менеджера нефтяной компании (несколько миллионов рублей), после чего разделили бы пополам. И потом бы всем говорили, что средняя зарплата в стране столько-то миллионов рублей.

    Но Вы же сами понимаете, что это не имеет смысла, потому что суммы на интервале распределены неравномерно. И в задаче про конверты тоже не оговорено распределение, поэтому некорректно предполагать, что оно будет равномерным.

    ОтветитьУдалить
  159. Анонимный23.09.2009, 23:52

    Здраствуйте!
    Не первый день пытаюсь понять, каким образом знание о содержимом первого конверта без знания о предельной сумме однораундного выигрыша влияет на распределение вероятности во втором.
    Конкретнее, каким образом вероятности уполовиненной и удвоенной суммы во втором конверте могут быть не равны.
    Если меньшую сумму обозначить - А, а открытую - Х, то вероятность того, что А=Х - 0,5 ; А=х\2 - 0,5. С каждым из этих сбытий жестко связано событие У - содержимое второго конверта. Вероятность А=Х влечет за собой А=У\2; А=Х\2 => А=У. То есть, возможные события во втором конверте равновероятны 0,5/0.5 , по моему.
    При этом, как не мудри,без знания предельной суммы и алгоритма генерирования А любая стратегия ведет к средневзвешенному однораундному выигрышу 1,5А .
    ------
    Спасибо.

    ОтветитьУдалить
  160. Уважаемый аноним, у Вас совершенно корректные выкладки: в самом деле, выбирая случайный первый конверт, мы получим A и 2A с равными вероятностями.

    Дальше Вы пишете «возможные события во втором конверте равновероятны», что тоже верно - во втором тоже будет А или 2А (и тоже с равными вероятностями). И с этим никто и не спорит.

    Вам осталось понять только одно утверждение: если в первом конверте X, а чему равно A мы не знаем, то во втором конверте будет Y=2X или Y=X/2, но мы не знаем, с какими вероятностями. Заметьте, что это утверждение очень отличается от Вашего о равновероятности событий Y=A и Y=2A.

    Как было сказано выше, это легко осознать, если понять, что нет равенства вероятностей следующих двух событий:
    1) в первом конверте X, причём X является меньшей суммой из двух сумм в конвертах,
    2) в первом конверте X, причём X является большей суммой из двух сумм в конвертах.

    Тонкость в том, что вероятности этих двух событий зависят от суммы X и от распределения случайной величины A.

    В статье http://www.maa.org/devlin/devlin_0708_04.html это показано очень доходчиво и весьма аккуратно. Успехов в освоении!
    Пишите, если останутся вопросы по этому парадоксу.

    Далее Вы пишете ложное утверждение «как не мудри,без знания предельной суммы и алгоритма генерирования А любая стратегия ведет к средневзвешенному однораундному выигрышу 1,5А», потому что ещё не поняли парадокса и предложенной стратегии. Когда разберётесь с первой частью, то мы сможем вернуться к этому вопросу (но я думаю, Вы всё поймёте к тому моменту). Кстати, в статье русскоязычной википедии появилось внятное объяснение выгодной стратегии - рекомендую его перечитать.

    ОтветитьУдалить
  161. Анонимный24.09.2009, 13:01

    Здравствуйте, Илья!
    Видимо, моя трудность заключается в попытке совместить теорию с реальностью.
    Вы пишете "мы не знаем, чему равно А"... Но мы договорились, что А -меньшая сумма из находящихся в паре конвертов ... Далее Вы
    рассматриваете уже другую величину(содержимое открытого конверта, по моему заранее введенному в рассуждение условию = Х) и говорите :"Мы не знаем, что в открытом конверте". Но мы договорились что в открытом конверте всегда Х !
    Мы не знаем коэффициента связи между А и Х .
    Видимо, именно на этом этапе и начинается "раздвоение сознания": Я решаю практическую или теоретическую задачу? "
    Хотелось бы найти такое решение, которое объединяет теорию и практику. На мой "дилетанский" взгляд, примеры того, что при некоторых значениях Х отрицание равновероятности удвоения- уполовинивания значения У относительно Х - практические.
    Но тогда "практичнее" сразу оговориться, что равновесие вариантов изначально кроется в величине А.
    Вмки я читал, проверю на наличие изменений.
    --------
    Спасибо.

    ОтветитьУдалить
  162. Уважаемый аноним, ответ на Ваш комментарий «Вы пишете "мы не знаем, чему равно А"... Но мы договорились, что А -меньшая сумма из находящихся в паре конвертов» состоит в следующем:
    Мы знаем, что A - меньшая сумма из двух сумм, находящихся в конвертах, но мы не знаем, чему она равна.

    Если X мы знаем (потому что пересчитали деньги в первом конверте), то A не знаем, а можем только догадываться, что A=X (если мы выбрали меньший конверт) или A=X/2 (если выбрали больший из двух конвертов).

    Разговоры об отличии теории и практики я не очень понимаю в данном контексте. Если есть утверждение "вероятности всегда равны 50%", то приведение корректного примера, когда это не так, автоматически опровергает это утверждение.

    Если кто-то скажет, что все лошади белые, то не надо ему доказывать, что природа допускает появление лошадей других цветов. Достаточно просто предъявить чёрную лошадь, так ведь? Это нормальный и корректных ход в доказательстве.

    Пишите, если у Вас остались вопросы после прочтения статьи http://www.maa.org/devlin/devlin_0708_04.html - разберёмся.

    ОтветитьУдалить
  163. Илья, я оставил бы вас (и себя) в покое с этими "распределениями вероятностей" и т.д. - мне это уже не интересно. Вам просто нужно утверждать, что вероятности вытащить в первом конверте бОльшую сумму (выиграть) и меньшую сумму (проиграть) тоже не равны. Тогда всё с вашей точно зрения будет понятно.

    Но спорить с тем, что выигрывая с первым конвертом и меняя его вы проигрываете (точно, 100%), а проигрывая с первым конвертом и безусловно меняя его вы точно выигрываете - глупо, т.к. таково условие задачи: в одном конверте проигрыш, в другом - выигрыш. И не важно, какая там сумма (замена-то безусловная). А вы именно с этим и спорите, отрицая равные вероятности выигрыша и проигрыша после безусловной замены конверта.

    Короче, так и останемся при своих. Извините, если что.

    ОтветитьУдалить
  164. D-nV, это называется не "останемся при своих", а "не хочу слышать".

    Вы пишете:
    Вам просто нужно утверждать, что вероятности вытащить в первом конверте бОльшую сумму (выиграть) и меньшую сумму (проиграть) тоже не равны

    Но мне это не нужно утверждать! Я неоднократно указывал, что это утверждение ложно, потому что эти вероятности равны.

    Далее Вы пишете:
    А вы именно с этим и спорите, отрицая равные вероятности выигрыша и проигрыша после безусловной замены конверта.

    Но я с этим не спорю. Вероятности выиграть и проиграть после безусловной замены конверта не меняются, оставаясь равными 50%.

    Если Вы хотите приписывать мне утверждения, с которыми я не согласен, то это Ваше право. Но не надо это называть честным спором. Вы общаетесь с самим собой, потому что спорите со своими словами (которые мне приписываете). Это грязная манипуляционная техника. Прекратите ей пользоваться, если хотите разобраться с этой задачей.

    ОтветитьУдалить
  165. Анонимный05.10.2009, 04:30

    Ответьте себе на вопрос, если мы возьмем случаи когда мы открыли конверт и там 10 рублей (без каких либо ограничений снизу и сверху и других) исключая случаи когда были первые конверты 50, 100 и тп. представьте себе 10 случаев мы берем 10 рублей, и 10 случаев берем следующий конверт, в первом случае 100 рублей, во втором...
    Фишка в том, что в качестве подарка мы получили неравномерное распределение по диапазонам значений. представьте себе ряд:
    1, 2, 4, 8, 32, 64 - грубо говоря это рост по экспоненте. То что в жизни такого не бывает нас и смущает, что вроде бы так не должно быть, что взяли 10, а реально это уже совсем не 10 а 12.5
    потому что событие "Б" КОНКРЕТНО ЗАВИСИМО.
    http://ru.wikipedia.org/wiki/%D0%9F%D0%B0%D1%80%D0%B0%D0%B4%D0%BE%D0%BA%D1%81_%D0%9C%D0%BE%D0%BD%D1%82%D0%B8_%D0%A5%D0%BE%D0%BB%D0%BB%D0%B0

    "Этот вывод противоречит интуитивному восприятию ситуации большинством людей, поэтому описанная задача и называется парадоксом Монти Холла."

    Поэтому нужно смотреть не на восприятие или ощущение, а на распределение.

    ОтветитьУдалить
  166. Анонимный05.10.2009, 06:54

    Илья, очевидно, что D-nV путает Ваши утверждения и утверждения авторов мембрановской статьи. Он спорит с Вами, потому что на мембране.ру нет возможности комментировать статью. Поэтому не надо ему отвечать.

    ОтветитьУдалить
  167. Александр15.10.2009, 18:43

    О чёёёммм ввыыы пиишшете?! Все только предпологают,а практикой никто не занялся.Смешно до истерики!ЖЕЛАЮ УДАЧИ!

    ОтветитьУдалить
  168. Анонимный31.10.2009, 16:36

    Цитата: "Теперь предположим, что распределение сумм в конвертах изначально неизвестно. Сумма выигрыша может составлять любое действительное число копеек, например, может быть меньше 1 копейки. В открытом конверте могут быть 100 рублей и 100 тысяч рублей с наперед неизвестной вероятностью.

    Однако случай, когда сумма в следующей игре не зависит от суммы в предыдущей, а все возможные суммы могут возникать с одинаковой вероятностью, математически невозможен."

    ОтветитьУдалить
  169. flyingru, всё верно. Выше мы как раз уже обсуждали невозможность некоторых типов случайных величин (рассматривали пример равномерной бесконечной случайной величины).

    ОтветитьУдалить
  170. Для тех, кто утверждает выгодность менять конверт.
    Введем второго виртуального игрока, который всегда будет выбирать оставшийся конверт, а затем обмениваться с первым игроком конвертами. Кто из них окажется в выигрыше (проигрыше)?

    ОтветитьУдалить
  171. prostachok,
    Вы, наверное, утверждаете, что менять конверт невыгодно.
    Тогда, предлагаю вам сыграть со мной по следующим правилам:
    1- Вы выбираете любую сумму в рублях и копейках для "первого конверта". И объявляете её публично. Я не знаю ни ваших желаний, ни ваших возможностей, поэтому эта сумма будет для меня совершенно непредсказуемой.
    2- Я, в соответствии со своими публичными обязательствами (офертой), заявляю, что предпочитаю другую сумму - "второго конверта".
    3- Попросим И. Весеннего случайным образом "фифти-фифти" выбрать дальнейшее решение - повышать или понижать сумму вдвое и также объявить об этом публично..
    4- Вы в соответствии с решением И. Весеннего объявляете сумму "второго конверта" и подсчитываете наш общий баланс: вы мне должны сумму "второго конверта", а я вам должен сумму "первого конверта" + комиссионые (10% от суммы "первого конверта" за вашу работу крупье).
    *- Ма-аленькое замечание. Чтобы у меня не возникло желания отказаться от выплаты из-за её дробности, прошу вас всегда назначать в "первый конверт" четную сумму. А комиссионные выплатим по итогам 10 игр.
    Сообщите заранее, приемлимы ли вам такие условия.

    ОтветитьУдалить
  172. eLectric, Вы осознанно шутите или искренне не поняли, что предлагаете совсем другую игру?

    По Вашим правилам менять конверт выгодно, а в обсуждаемой в этой заметке игре этого нет.

    ОтветитьУдалить
  173. eLectric
    Предложена совсем другая игра. Я хотел подчеркнуть, что утверждение выгодноси всегда менять конверт, сомнительна. Мое личное убеждение, для одиночного выбора есть смысл выбрать второй конверт, а вот для серии выборов придерживаться правила:
    сумму денег всех вскрытых конвертов разделить на количество конвертов и умножить на 1,25, полученное значение является границей для принятия решения. Если в конверте денег меньше, то менять в противном случае, не менять. Вот и все.

    ОтветитьУдалить
  174. [b]Илья Весенний, [/b] Я искренне не понял, что предлагаю другую игру. Я правильно понял различие: "Предлагаются на выбор два уже заполненных конверта" против "Второй конверт заполняется после просмотра первого"?
    Я искренне считаю, что поскольку просмотр содержимого первого конверта, по моим правилам, не влияет на наполнение второго конверта, то оба способа эквивалентны.
    Я согласен с вами, что мой пост можно назвать осознанной шуткой и не согласен с вами, что "По Вашим правилам менять конверт выгодно".
    Серьезно говоря, я думаю, что эти вызывающие правила не принесут выйгрыша игроку, а только потери с учетом комиссионных.
    Мне кажется, что при разных вариантах игры (предлагать сразу два конверта либо по-очереди) видится, что существенно отличаются рассуждения, хотя, по идее, и приводят к одинаковым результатам.

    [b]prostachok, [/b]
    Мне кажется, что годится для одиночного выбора, может долго годится и для серии. Крупье может применять такую тактику: искать спонсора случайными и разнообразными способами. Просить копейки у прохожих, тысячи рублей у родственников, кредиты в банке. Так что игрок долго не сможет ориентироваться на какой-то предел.
    Тогда правильно было-бы исходить из неопределенности предела (но не бесконечности). Но тут-уж теряюсь, как считать вероятности

    ОтветитьУдалить
  175. prostachok, интересные вещи Вы говорите: «для одиночного выбора есть смысл выбрать второй конверт»... Мы же вроде бы выше подробно разобрали эту задачу.

    eLectric, здесь работают обычные html-тэги, поэтому не надо использовать bbcode.

    В заметке и комментариях выше мы разобрались, чем отличаются эти две игры. Я не вижу смысла повторять те абзацы ещё раз, но готов ответить на вопросы, если они останутся у Вас после прочтения. Кстати, рекомендую прочитать статью википедии на эту тему (ссылка на неё есть в заметке) - возможно, там написано понятнее.

    ОтветитьУдалить
  176. Илья
    Спасибо за замечание, ознакомился с комментариями более внимательно. Насколько я понял, пародокс состоит в том, что идет подмена условия задачи, и уже теорию этой задачи применяют к первоначальной.
    1. исходные условия - максимальная и минимальная сумма содержимого коверта может оказаться как в первом так и во втором конвертах.
    2. второй конверт формирутся после первого - максимальная и минимальные суммы могут оказаться только во втором конверте, что и приводит к выигрышности всегда брать второй.
    3. если сумма х может достигать значения inf, то 2*х значения 2*inf. Отсюда, если в 1 конверте сумма > inf, во втором конверте со сто процентной вероятностью меньшая сумма и с нулевой вероятностью, бOльшая... Соответственно распределение вероятностей выпадания во втором конверте 2х и х/2 неравномерное.
    Если я правильно понял, вокруг этого вся возня и затеяна.
    Спасибо тебе Илья, все было довольно любопытно.

    ОтветитьУдалить
  177. Илья Весенний,
    Да, да, вы правы, это я тормознул.
    Меня вдруг смутило, что в любом варианте раскладов во втором конверте в среднем 1,25 от первого.

    ОтветитьУдалить
  178. prostachok, полагаю, Вы почти всё поняли. Меня только очень смущает третий пункт Вашего комментария: понятие бесконечность достаточно сложное, поэтому лучше аккуратнее делать такие высказывания.

    eLectric, здорово, что разобрались! :)

    ОтветитьУдалить
  179. Илья
    По поводу осторожного обращения с бесконечностью, у меня все корректно, так как это одна и таже бесконечность, она лежит в конверте с меньшей суммой.

    Кстати, ваши утверждения от 04.09.09 10:19
    Ещё раз перечислю верные утверждения:
    1) вероятность того, что в первом конверте будет сумма большая из двух (из тех двух, что лежат в конвертах) - 50%.
    2) вероятность того, что во втором конверте обнаружется сумма меньшая из двух - 50%.
    3) вероятность того, что если в первом конверте x, то во втором будет 2x - какая угодно (это зависит от закона распределения денег по конвертам и от значения x)

    не совсем точны. В часности, сумма вероятностей выбора второго конверта с меньшей или бОльшей суммой равна единице. Отсюда - вероятность выбора второго конверта с меньшой (бОльшей) суммой какая угодно и зависит от закона распределения денег по конвертам и значения х.

    Интуитивно чувствуя это мною была предложена стратегия игры (от 06.11.09 17:11)
    Мое личное убеждение, для одиночного выбора есть смысл выбрать второй конверт, а вот для серии выборов придерживаться правила:
    сумму денег всех вскрытых конвертов разделить на количество конвертов и умножить на 1,25, полученное значение является границей для принятия решения. Если в конверте денег меньше, то менять в противном случае, не менять.

    Кстати, множитель 1,25 получен как (х/2 + 2х)/х. Я просимулировал эту игру в среде Excel и получил положительный результат, но еще значительней результат был с коэффициентом 1,5 (х + 2х)/х. При бОльшем диапозоне значений и количестве попыток выигрыш стабильно приближался к значению 1,25.

    ОтветитьУдалить
  180. prostachok, в той математике, которую изучал и изучаю я, бесконечность не является числом. Поэтому мне трудно понять, что значит фраза «она лежит в конверте с меньшей суммой». Она там не может лежать, так как сумма денег в конверте (в моём понимании) является числом.

    Или объясняйте, что значат все эти термины в вашем представлении о математике, или используйте их классические значения. Иначе мы не будем друг друга понимать.

    ОтветитьУдалить
  181. Анонимный10.11.2009, 09:30

    Good day, Iliya.

    The only statement of yours I wouldn't agree with is when you say: This paradox is a fraud. The problem here is not to prove that exchanging your envelope can't be always advantageous - everybody with a modicum of common sense understands that's not the case. The challenge of the exchange paradox is to find the flaw behind the reasoning that leads one to believe that expected gain on trading your envelope for the unopened one is always positive.

    It is clear to me that you know where the reasoning went wrong. That's why I would like to offer you the following modification of the problem.

    The original exchange problem has a major disadvantage - it is meaningless as a game of competition between a casino and a player. Such a game, plainly, could never be offered in a gambling house. Gambling houses are not charities - they are in the business of making money, not giving it away for free. But it takes just a tiny tweak in the statement of the problem to breathe a new life into it and make it a vibrant, meaningful game of competition between the house and the player.

    Here is my version of the problem.

    In a gambling house, the player is offered two sealed envelopes each containing a bank cheque; one cheque is twice the value of the other. The player may open one of the envelopes and observe the cheque in it. Then a decision is to be made by the player - which envelope he or she wishes to buy. Face-value of the cheque (which can be any real number in the continuous interval x>0) in one envelope sets the price for the other envelope.

    The goal in this zero-sum game for both adversaries is to maximize their respective gains. The perfect symmetry of the game suggests that buying the opened envelope at each round as well as its mirror strategy - always buying the closed one - is a trivial Nash equilibrium solution. Are there any other meaningful strategies in behalf of the player or the house?

    Unfortunately I do not have a keyboard with Cyrillic. Besides, I am a very bad typist and it would take me forever to write this in Russian. But I do speak Russian, so you can reply in Russian. This way everybody on your blog can participate in solving this problem.

    Best regards,
    Arthur

    ОтветитьУдалить
  182. Анонимный10.11.2009, 11:36

    Доброго времени суток.
    Прочитал обсуждения, согласен с точкой зрения автора. Однако ничуть не порадовала статья из википедии: невозможность получения случайного числа из промежутка от 0 до бесконечности (равномерного распределения чисел на бесконечном интервале) явно в статье не указана. Так же не понял следующих слов:
    цитата: "Например, если предположить, что сумма в меньшем конверте равномерно распределена между 10 руб. и 100 руб., очевидно такое:
    Если выпало 15 руб., в другом конверте определённо 30 руб. (7,5 быть не может). Поэтому требуется брать другой конверт.
    Если выпало 25 руб., в другом может быть как 12,5 руб., так и 50 руб., но вероятность второго больше — желательно брать другой конверт.
    Если выпало 90 руб., в другом может быть как 45 руб., так и 180 руб., при этом больше вероятность первого — лучше не брать другой конверт.
    Если выпало 150 руб., в другом конверте определённо 75 рублей — ни в коем случае не брать другой конверт." - википедия.

    Поленился считать для 10-100, посчитал для 0-100. Вышло вот что:
    в 25% - в конверте 100-200 руб => доход от обмена конверта -0.5.
    в 75% - в конверте 0-100 руб, причем в трети случаев - это конверт с большей суммой, т. е. доход от обмена конверта -.5. В остальных 2\3 (50% от общего количества игр) в конверте меньшая сумма: доход от обмена =+1. В итоге, если в конверте 0-100 рублей (75% игр) доход от обмена 1\3*(-0.5)+2\3*(1)=1\2. Т. е. для данных 75% случаев доход от обмена (в среднем) ВСЕГДА (на всем промежутке от 0 до 100) =150%. В приведенной мною цитате из википедии при сумме в конверте =90 р советуют воздержаться от обмена, мотивируя это тем что, "при этом больше вероятность первого — лучше не брать другой конверт". Я же полагаю, что вероятность на ВСЕМ отрезке одинаковая. С цифрами 10-100 сложнее только расчеты, там тоже будет зона 100-200 (при обмене всегда проигрываем), зона 10-20 (меняем - всегда выигрываем), зона 20-100 (выигрываем при обмене в большинстве случаев, шанс выйгрыша постоянный, считать уныло). По этим же расчетом в цитате: "Также они открыли, что предопределённый обмен, когда выбирается альтернативный конверт только в том случае, если сумма в первом меньше заранее и наугад выбранного игроком значения, тоже работает, хотя этот вывод кажется абсолютно противоестественным, поскольку о минимальной планке перевыбора конвертов знает лишь игрок, но не те, кто предположительно эти деньги туда кладёт."
    - явная ложь, потому что выбранное число должно ОБЯЗАТЕЛЬНО быть меньше верхнего интервала выборки случайных чисел для сумм(количества денег) в меньшем из конвертов. Понятно, что при компьютерной симуляции верхний интервал рандома огромен, а значения выбирают люди, но мне в глаза бросаются эти строки как явная ошибка (интуитивно и при расчетах).
    Английскую статью к сожалению не смотрел, иностранным владею слабо.
    Очень прошу хозяина блога объяснить, где я был неправ в своих рассуждениях или (а вдруг), если я прав дать ссылку (по возможности) на ресурс (по возможности русскоязычный), где этот вопрос решается так же как сделал я.

    З.Ы. прочитал там же пару строк про пародокс Паррондо - просто потрясло, если есть, дайте пожалуйста ссылки на грамотное описание его (желательно применительно к покеру - смахивает на прогресс в теории)

    ОтветитьУдалить
  183. Arthur, спасибо за интересную версию задачи. Я подумаю о ней и попробую найти статьи на эту тему. Предложенная Вами вариация очень естественна, поэтому, скорее всего, о ней уже многие думали.

    Уважаемый аноним, благодарю за интересные возражения к статье википедии. Надо будет подумать и поправить её так, чтобы подобных вопросов не могло возникнуть.

    Что касается парадокса Паррондо, то хороших материалов на русском я не видел, а на английском, как я понял, Вам пока не подойдёт. Если смогу найти, то напишу здесь же комментарий со ссылками.

    ОтветитьУдалить
  184. Уважаемый Илья! Вчера прочитал Вашу заметку к статье на мембране, а так же все комметарии в данной ветке и хочу предложить к обсуждению свой взгляд на поднятую Вами проблему.
    Прежде всего мне представляется возможным и необходимым разделить понятия "вероятности распределения сумм в оставшемся (после вскрытия первого) втором конверте" и "стратегии игры". В настоящем посте я буду вести речь исключительно о вероятности.

    Исходные условия.
    1.Играют двое: Генератор Случайных Чисел (распределяет числа по конвертам) и Я (Мы :).
    2.Играют не на деньги, а на числа.
    3.Интервал возможных используемых целых чисел (N) - конечен: от 1 до n.

    Игра.
    В первом выбранном и вскрытом нами случайным образом конверте обнаруживается некое число Х. Нас интересует вопрос: с какой вероятностью в оставшемся (втором) конверте может оказаться число 2Х и с какой вероятностью там же может оказаться число Х/2.
    Для ответа на этот вопрос я предлагаю условно разделить интервал всех возможных значений суммы во втором конверте на две части: (от 1 до n/2 - здесь все целые числа) и (от n/2+1 до n - здесь исключительно четные числа).
    Тогда вероятность попадания числа Х во вскрытом нами первом конверте изначально известного нам конечного интервала N (независимо от того - вскрыли мы конверт или нет) составит:
    - для второго интервала - 1/3;
    - первого интервала - 2/3.
    Далее. Еесли число Х в первом конверте попадает в интервал (n/2 - n), то с вероятностью 100% (или 1/3 всех возможных выриантов из интервала N) число во втором конверте составит Х/2.
    Если же число Х в первом конверте попадает в интервал (1 - n/2), то происходит следующее:
    - если число Х нечетное, то с вероятностью 100% (или 1/3 всех возможных выриантов из интервала N) число во втором конверте составит 2Х;
    - если число Х четное, то с вероятностью 50% (или 1/6 всех возможных выриантов из интервала N) число во втором конверте может составить как Х/2, так и 2Х.
    Таким образом, я попытался показать, что независимо от того, какое число в первом конверте, вскрыли мы конверт или нет, вероятность того, что во втором конверте окажется число Х/2 или 2Х составляет 1/2, или 50х50.

    С уважением.

    P.S.В данном моем посте речь шла исключительно о вероятности событий. Другое дело - стратегия игры. Но об этом в другой раз (если конечно это уместно в данной ветке)

    ОтветитьУдалить
  185. Smith, я понимаю Ваш комментарий, но не вижу, как получился финальный аккорд: «независимо от того, какое число в первом конверте, вскрыли мы конверт или нет, вероятность того, что во втором конверте окажется число Х/2 или 2Х составляет 1/2, или 50х50».

    Что вообще значит эта фраза? Вы же только что выше сами написали, что вероятность зависит от X (например, от чётности этого X, от того, как соотносятся X и n)... Что Вы хотели сказать последним утверждением? Я не понимаю :(

    ОтветитьУдалить
  186. Спасибо, Илья, Вы правы. В оригинале этот обзац звучал так:
    "...независимо от того, какое число в первом конверте, вскрыли мы конверт
    или нет, ПРИ ДАННЫХ МНОЮ ИСХОДНЫХ УСЛОВИЯХ вероятность того, что, во
    втором конверте окажется число Х/2 или 2Х составляет 1/2, или 50х50."
    Это был один из ключевых моментов поста - показать, что при определенных
    условиях такое вполне возможно. Однако произошел сбой, и я потерял концовку текста, так что дописывал уже на скорую руку, и - вот результат.
    Прошу Вас, если это возможно, внести соответствующее изменение в
    предыдущий пост.
    Благодарю за понимание.

    ОтветитьУдалить
  187. Smith, здорово, что мы друг друга понимаем!
    Увы, комментарии здесь редактировать невозможно, поэтому не смогу помочь.
    Продолжение темы можно прочитать в новой заметке.

    ОтветитьУдалить
  188. Анонимный01.02.2010, 19:18

    Здравствуйте.
    Илья, парадокс Паррондо Вы не разбирали? Существует ли на самом деле парадокс? Или, в действительности, имеется заявленный как парадокс эффект (но таковым не являющийся)- при комбинации двух заведомо проигрышных игр возможно получить выигрышную?
    С уважением, sts.
    PS большое спасибо за блог и возможность общаться с интересными людьми.

    ОтветитьУдалить
  189. sts, спасибо за тёплые слова!

    Давайте разберёмся с терминологией. Что такое вероятностный парадокс? Это такая задачка, про которую интуиция (даже подкреплённая какими-то знаниями теории) подсказывает один ответ, а честное решение оказывается совсем другим. То есть, мы имеем расхождение обыденного представления о вероятностях и научного. Не более того.

    В этом смысле, парадокс Паррондо выглядит очень даже парадоксально для неспециалиста. Бытовая интуиция не понимает, как так может быть. Грамотная комбинация проигрышных игр, конечно, может быть выигрышной. На эту тему хватает примеров.

    И Вы правы, лучше было называть такие вещи не парадоксами, а эффектами... Это уменьшило бы путаницу, наверное. Но сейчас уже лучше пользоваться устоявшейся терминологией, потому что иначе есть большой риск не быть понятым.

    ОтветитьУдалить
  190. Анонимный26.02.2010, 09:36

    не вполне понимаю, зачем столько сложностей.

    1). пока оба конверта запечатаны, вероятность взять конверт с суммой х равна 50%. значит, вероятность взять конверт с суммой 2х равна 100-50=50%.

    2). сам процесс замены конверта являет собой ни что иное кроме как перевычитание вероятностей. НО если обе вероятности изначально равны 50% - фактически, для исхода (в плане ВЕРОЯТНОСТИ) процесс замены конверта значения не имеет...

    Следствие: если *в плане вероятности* замена значения не имеет, то и стратегию игры выработать невозможно..

    ОтветитьУдалить
  191. Уважаемый аноним, смысл сложностей в следующем: для данной задачи Вам очевидно, что надо рассуждать так-то и так-то, тогда всё будет корректно. Но для другой задачи есть риск поверить красивым наукообразным словам, что может привести к неверному принятию решения.

    Поэтому надо уметь не только придумать своё правильное решение, но и найти ошибку в чужом. Причём не просто найти, но ещё и доходчиво показать, что эта ошибка критичная, а не "мелочь".

    ОтветитьУдалить
  192. Анонимный27.02.2010, 06:34

    Илья, I've wasted quite some time thinking about this problem. I hope you and your readers will find my thoughts about the subject at least entertaining:

    Game Strategies in the Exchange Problem

    Best regards,
    Arthur

    ОтветитьУдалить
  193. Анонимный28.02.2010, 09:14

    Илья, yes, the link is broken indeed. I could email the pdf file to you but I do not see any email addresses on your web site. Drop me a note at arthur.baraov@hp.com and I would be happy to reply with the the paper attached.

    ОтветитьУдалить
  194. Семён Стяжкин18.04.2010, 01:25

    замена конвертов смысла не имеет.если сумму в первом конверте принять за х.то в другом конверте будет 2х или 0.5х.А среднее геометрическое между ними=корнеь(2х*0.5х)=х.в среднем сумма везде одинаковая,замена смысла не имеет.

    ОтветитьУдалить
  195. Семён, а почему рассматривать следует именно среднее геометрическое?

    Представьте, что в одном конверте нет ничего (0 рублей), а в другом какая-то сумма (например, 100 рублей). Если Вы много раз будете выбирать случайный из этих двух конвертов, то в среднем за каждую попытку получите 50 рублей, верно?

    А формула среднего геометрического для пары из чисел 0 и 100 всегда даст 0.

    ОтветитьУдалить
  196. Семён Стяжкин18.04.2010, 13:26

    но если в первом конверте ноль,то разумно выбирать второй конверт.и вообще ноль быть не может,по условию во втором конверте тогда будет 0*2=0.или 0:2=0.

    ОтветитьУдалить
  197. Семён Стяжкин18.04.2010, 13:43

    всё верно.Если в первом конверте 0, тогда во втором 0*2=0 или 0:2=0 по условию задачи.если в первом ничего нет, то и во втором тоже ничего нет в любом случае-среднее геометрическое дает верный ответ=0.А в той задаче которую вы предложили, если мне попадется конверт где ничего нет,я сразу выберу другой конверт.

    ОтветитьУдалить
  198. Семён, в моём предыдущем комментарии была предложена другая задачка для иллюстрации мысли - в ней герой выбирает один из двух случайных конвертов, на чём игра и прекращается. Но давайте пока от неё отвлечёмся.

    Объясните, пожалуйста, почему среди всех возможных средних величин (среднее арифметическое, среднее гармоническое и так далее) следует выбирать именно среднее геометрическое? Как это доказать?

    ОтветитьУдалить

Понравилась заметка? Подпишитесь на RSS-feed или email-рассылку.

Хотите поделиться ссылкой с другими? Добавьте в закладки:



Есть вопросы или предложения? Пишите письма на адрес mytribune АТ yandex.ru.

С уважением,
      Илья Весенний